Re: [R] Warning message

2010-05-10 Thread Dennis Murphy
Hi:

A univariate t-distribution with 1 df is equivalent to the standard Cauchy
distribution, for
which it is well established that the population mean does not exist. You're
basically
simulating a vector version of the Cauchy distribution (an IID 8-dimensional
version),
so the same problem is likely to arise. This is exhibited in the tail
behavior of the
(simulated) distribution of means.

It's a little more convenient to do this in terms of arrays rather than
lists:

library(mvtnorm)

# Generate 10 multivariate t samples in 8D and replicate the process 1000
times
# result is a 3D array of size 10 x 8 x 1000.
rs - array(replicate(1000, rmvt(10, sigma = diag(8), df = 1)), c(10, 8,
1000))

# Average over the 10 samples in each replicate, yielding an 8 x 1000 matrix
-
# i.e., average over the first dimension of the array
rsm - apply(rs, c(2, 3), mean)

# Transpose the matrix, convert it to a data frame, and melt it using the
# reshape package; this generates an 8000 x 2 data frame, where the first
# variable is an indicator for each component of the vector (V1 - V8) and
# the value represents the sample values.
library(reshape)
rr - melt(as.data.frame(t(rsm)))

# Use the histogram function in lattice to generate the histograms for each
# component of the sample vector:
library(lattice)
histogram(~ value | variable, data = rr, xlab = 'x', type = 'density')

Since your original post averaged over the entire matrix in each replicate,
let's do that - just because...

rmns - apply(rs, 3, mean)
hist(rmns, breaks = 20, probability = TRUE)
#or in lattice,
histogram(~ rmns, breaks = 20, density = TRUE)

[Hint: If you use the median rather than the mean as an estimator, its
 sampling distribution will behave somewhat closer to what you
 may expect...]

HTH,
Dennis

On Sun, May 9, 2010 at 10:37 PM, Shant Ch sha1...@yahoo.com wrote:

 Hello,

 I want to draw a histogram of the mean of sample observations drawn from
 multivariate t distribution. I am getting the following error corresponding
 to the code I used. Though I am getting the graph, but I am curious to know
 the warning message.

 Warning messages:
 1: In if (freq) x$counts else { :
  the condition has length  1 and only the first element will be used
 2: In if (!freq) Density else Frequency :
  the condition has length  1 and only the first element will be used

 ---
 library(mvtnorm);
 s12-c(1:1000);
 var21-lapply(s12,function(x){
 rs-rmvt(10, sigma=diag(8), df=1);
 my-mean(rs);
 sy-sqrt(var(rs))
 return(cbind(my,sy))
 });
 data1-do.call(rbind,var21);
 dataMat-data.frame(data1);
 W-dataMat$my;
 hist(W,breaks=20,probability=T)

 ---

 Thanks.

 Shant




[[alternative HTML version deleted]]

 __
 R-help@r-project.org mailing list
 https://stat.ethz.ch/mailman/listinfo/r-help
 PLEASE do read the posting guide
 http://www.R-project.org/posting-guide.html
 and provide commented, minimal, self-contained, reproducible code.


[[alternative HTML version deleted]]

__
R-help@r-project.org mailing list
https://stat.ethz.ch/mailman/listinfo/r-help
PLEASE do read the posting guide http://www.R-project.org/posting-guide.html
and provide commented, minimal, self-contained, reproducible code.


Re: [R] Warning message

2010-05-10 Thread Peter Dalgaard
Shant Ch wrote:

 Warning messages:
 1: In if (freq) x$counts else { :
   the condition has length  1 and only the first element will be used
...
 hist(W,breaks=20,probability=T)

A variable called T lying around with length(T) != 1 ?

Try probability=TRUE instead.

-- 
Peter Dalgaard
Center for Statistics, Copenhagen Business School
Phone: (+45)38153501
Email: pd@cbs.dk  Priv: pda...@gmail.com

__
R-help@r-project.org mailing list
https://stat.ethz.ch/mailman/listinfo/r-help
PLEASE do read the posting guide http://www.R-project.org/posting-guide.html
and provide commented, minimal, self-contained, reproducible code.


[R] How to identify seasonality in time series data??

2010-05-10 Thread vikrant

Dear ALL R Experts,

I have a time series and I want to detect presence of seasonality in this
time series. I know the method of plotting the Autocorrelation function(acf)
and if there are significant lags after period s,2s,3s,... then s is the
period for the time series. But my problem is instead of plotting the ACF
function, I need to automatically test the presence of seasonality. How this
could be done??

Since I have many time series, I need to automate this function which would
detect the presence of seasonality along with the period. Is there any
package readily available which could do this?? Or there is anyway to do
this. Any help would e greatly appreciated



Thanks in advance

-- 
View this message in context: 
http://r.789695.n4.nabble.com/How-to-identify-seasonality-in-time-series-data-tp2165031p2165031.html
Sent from the R help mailing list archive at Nabble.com.

__
R-help@r-project.org mailing list
https://stat.ethz.ch/mailman/listinfo/r-help
PLEASE do read the posting guide http://www.R-project.org/posting-guide.html
and provide commented, minimal, self-contained, reproducible code.


[R] how to get p-value from ave

2010-05-10 Thread Yuan Jian
Hi there,

I checked google for aov. usually one uses summary to see whether the p-value 
is small.
but I want to put aov in my script. how can I get the p-value, (0.1115, 0.6665, 
0.6665 in the following example)?

thanks
YU



 datafilename=http://personality-project.org/r/datasets/R.appendix2.data;
 data.example2=read.table(datafilename,header=T)
 aov.ex2 = aov(Alertness~Gender*Dosage,data=data.example2) 
 summary(aov.ex2)
  Df  Sum Sq Mean Sq F value Pr(F)
Gender 1  76.562  76.562  2.9518 0.1115
Dosage 1   5.062   5.062  0.1952 0.6665
Gender:Dosage  1   0.063   0.063  0.0024  0.6665
Residuals 12 311.250  25.938   




  
[[alternative HTML version deleted]]

__
R-help@r-project.org mailing list
https://stat.ethz.ch/mailman/listinfo/r-help
PLEASE do read the posting guide http://www.R-project.org/posting-guide.html
and provide commented, minimal, self-contained, reproducible code.


Re: [R] how to get p-value from ave

2010-05-10 Thread Dennis Murphy
Hi:

str() is usually helpful:
 str(summary(aov.ex2))
List of 1
 $ :Classes ‘anova’ and 'data.frame':   4 obs. of  5 variables:
  ..$ Df : num [1:4] 1 1 1 12
  ..$ Sum Sq : num [1:4] 76.5625 5.0625 0.0625 311.25
  ..$ Mean Sq: num [1:4] 76.5625 5.0625 0.0625 25.9375
  ..$ F value: num [1:4] 2.95181 0.19518 0.00241 NA
  ..$ Pr(F) : num [1:4] 0.111 0.666 0.962 NA
 - attr(*, class)= chr [1:2] summary.aov listof

summary.lm() outputs a list object with one component and five
subcomponents.  [[1]] extracts the first component of the list;
[[1]][5] extracts the fifth subcomponent.

This suggests either of the following ways to extract the p-values:

# By component number
 summary(aov.ex2)[[1]][5]
  Pr(F)
Gender0.1114
Dosage0.6665
Gender:Dosage 0.9617
Residuals

# By name
 summary(aov.ex2)[[1]]$'Pr(F)'
[1] 0.1114507 0.6664956 0.9616567NA

HTH,
Dennis

On Mon, May 10, 2010 at 12:17 AM, Yuan Jian jayuan2...@yahoo.com wrote:

 Hi there,

 I checked google for aov. usually one uses summary to see whether the
 p-value is small.
 but I want to put aov in my script. how can I get the p-value, (0.1115,
 0.6665, 0.6665 in the following example)?

 thanks
 YU



  datafilename=http://personality-project.org/r/datasets/R.appendix2.data
 
  data.example2=read.table(datafilename,header=T)
  aov.ex2 = aov(Alertness~Gender*Dosage,data=data.example2)
  summary(aov.ex2)
   Df  Sum Sq Mean Sq F value Pr(F)
 Gender 1  76.562  76.562  2.9518 0.1115
 Dosage 1   5.062   5.062  0.1952 0.6665
 Gender:Dosage  1   0.063   0.063  0.0024  0.6665
 Residuals 12 311.250  25.938





[[alternative HTML version deleted]]


 __
 R-help@r-project.org mailing list
 https://stat.ethz.ch/mailman/listinfo/r-help
 PLEASE do read the posting guide
 http://www.R-project.org/posting-guide.html
 and provide commented, minimal, self-contained, reproducible code.



[[alternative HTML version deleted]]

__
R-help@r-project.org mailing list
https://stat.ethz.ch/mailman/listinfo/r-help
PLEASE do read the posting guide http://www.R-project.org/posting-guide.html
and provide commented, minimal, self-contained, reproducible code.


[R] r matrix inconsistencies?

2010-05-10 Thread Seth


-- 
View this message in context: 
http://r.789695.n4.nabble.com/r-matrix-inconsistencies-tp2165121p2165121.html
Sent from the R help mailing list archive at Nabble.com.

__
R-help@r-project.org mailing list
https://stat.ethz.ch/mailman/listinfo/r-help
PLEASE do read the posting guide http://www.R-project.org/posting-guide.html
and provide commented, minimal, self-contained, reproducible code.


Re: [R] median of two groups

2010-05-10 Thread cheba meier
Thank you very much for that.

What is then if I have unpaired and unbalanced samples?

Best regards,
Cheba

2010/5/7 Bert Gunter gunter.ber...@gene.com

 Perhaps this might help clarify:

 sample A:   10   15   20
 sample B:   12   15   22

 Median of sample A = 15; median of sample B = 15. Sample medians are =.
 But: B-A differences are 2,0,2 with a median of 2. So the median difference
 does not equal the difference of the medians.

 Clarity in what you wish to test (and why!) is essential to determine how.


 Bert Gunter
 Genentech Nonclinical Statistics

 -Original Message-
 From: r-help-boun...@r-project.org [mailto:r-help-boun...@r-project.org]
 On
 Behalf Of cheba meier
 Sent: Friday, May 07, 2010 12:46 PM
 To: Joris Meys
 Cc: R-help@r-project.org; Thomas Lumley
 Subject: Re: [R] median of two groups

 Hi all,

 Thank you for your reply.

 if done properly! What does this mean? The R-code I have is using the
 R-function sample without replacement. Am I doing this properly?

 median of the differences is zero! Does this mean if I run 1000 permutation
 and for each permutation I compute the median difference and as a result I
 have 1000 differences. Is the the H0: median(1000 differences) =0? If yes,
 which conclusion one would have from this H0?

 Best wishes,
 Cheba



 2010/5/7 Joris Meys jorism...@gmail.com

  depends on how you interprete absolute median difference. Is that the
  absolute difference of the medians, or the median of the absolute
  differences. Probably the latter one, so you would be right. If it's the
  former one, then it is testing whether the difference of the medians is
  zero.
 
  Cheers
  Joris
 
 
  On Fri, May 7, 2010 at 6:52 PM, Thomas Lumley
 tlum...@u.washington.eduwrote:
 
  On Fri, 7 May 2010, cheba meier wrote:
 
   Dear Thomas,
 
  I have been running simulations in order me to understand this problem!
 I
  have found something online where the absolute median difference is
  computed
  and permutations are ran to compute a p-value. Is such a test (if I can
  call
  it a test) tests the null hypothesis that median group 1 = median group
  2?
 
 
  No, that is testing whether the median of the differences is zero.  This
  is not the same as testing whether the difference of the medians is
 zero.
 
 -thomas
 
 
 
   Thank you in advance for your help.
 
  Regards,
  Cheba
 
  2010/4/6 Thomas Lumley tlum...@u.washington.edu
 
 
 
  None of them.
 
   - mood.test() looks promising until you read the help page and see
 that
  it
  does not do Mood's test for equality of quantiles, it does Mood's test
  for
  equality of scale parameters.
   - wilcox.test() is not a test for equal medians
   - ks.test() is not a test for equal medians.
 
 
  Mood's test for the median involves dichotomizing the data at the
 pooled
  median and then doing Fisher's exact test to see if the binary
 variable
  has
  the same mean in the two samples.
 
  median.test-function(x,y){
   z-c(x,y)
   g - rep(1:2, c(length(x),length(y)))
   m-median(z)
   fisher.test(zm,g)$p.value
  }
 
  Like most exact tests, it is quite conservative at small sample sizes.
 
 -thomas
 
 
  On Tue, 6 Apr 2010, cheba meier wrote:
 
   Dear all,
 
 
  What is the right test to test whether the median of two groups are
  statistically significant? Is it the wilcox.test, mood.test or the
  ks.test?
  In the text book I have got there is explanation for the Wilcoxon
 (Mann
  Whitney) test which tests ob the two variable are from the same
  population
  and also ks.test!
 
  Regards,
  Cheba
 
[[alternative HTML version deleted]]
 
  __
  R-help@r-project.org mailing list
  https://stat.ethz.ch/mailman/listinfo/r-help
  PLEASE do read the posting guide
  http://www.R-project.org/posting-guide.html
  and provide commented, minimal, self-contained, reproducible code.
 
 
   Thomas Lumley   Assoc. Professor, Biostatistics
  tlum...@u.washington.eduUniversity of Washington, Seattle
 
 
 
 [[alternative HTML version deleted]]
 
  __
  R-help@r-project.org mailing list
  https://stat.ethz.ch/mailman/listinfo/r-help
  PLEASE do read the posting guide
  http://www.R-project.org/posting-guide.html
  and provide commented, minimal, self-contained, reproducible code.
 
 
  Thomas Lumley   Assoc. Professor, Biostatistics
  tlum...@u.washington.eduUniversity of Washington, Seattle
 
  __
  R-help@r-project.org mailing list
  https://stat.ethz.ch/mailman/listinfo/r-help
  PLEASE do read the posting guide
  http://www.R-project.org/posting-guide.html
  and provide commented, minimal, self-contained, reproducible code.
 
 
 
 
  --
  Joris Meys
  Statistical Consultant
 
  Ghent University
  Faculty of Bioscience Engineering
  Department of Applied mathematics, biometrics and process control
 
  Coupure Links 653
  B-9000 

Re: [R] update R 2.11.0,there is error when usi ng plot(), how can I do?

2010-05-10 Thread Uwe Ligges



On 09.05.2010 18:37, bbslover wrote:



a-1:5
b-2:6
plot(a,b)

Error in function (width, height, pointsize, record, rescale, xpinch,  :
   Graphics API version mismatch


before, R 2.10  , plot() is ok.   Now, R 2.11.0 does not work



YYou have probably an old version  of the graphics package in your 
search path.
Just look into your libraries and delete any graphics package that are 
not in your R_HOME/library


Uwe Ligges

__
R-help@r-project.org mailing list
https://stat.ethz.ch/mailman/listinfo/r-help
PLEASE do read the posting guide http://www.R-project.org/posting-guide.html
and provide commented, minimal, self-contained, reproducible code.


[R] scope of a function + lazy evaluation

2010-05-10 Thread sayan dasgupta
Hey guys,

I have a doubt here , It is something simple I guess, what am I missing out
here ??


f - function(y) function() y
tmp - vector(list, 5)
for (i in 1:5) tmp[[i]] - f(i)
tmp[[1]]() # returns 5;

z - f(6)
tmp[[1]]() # still returns 5; it should return 6 ideally right ???

Even if  I dont evaluate the function tmp[[1]] before i.e I do
rm(list=ls())
f - function(y) function() y
tmp - vector(list, 5)
for (i in 1:5) tmp[[i]] - f(i)
 z - f(6)
tmp[[1]]() # it still returns 5; it should return 6 ideally right ???

[[alternative HTML version deleted]]

__
R-help@r-project.org mailing list
https://stat.ethz.ch/mailman/listinfo/r-help
PLEASE do read the posting guide http://www.R-project.org/posting-guide.html
and provide commented, minimal, self-contained, reproducible code.


Re: [R] tapply function with NA

2010-05-10 Thread Gabor Grothendieck
See ?colSums

On Mon, May 10, 2010 at 12:44 AM, vincent.deluard
vincent.delu...@trimtabs.com wrote:

 Hi R users,

 I have a matrix m of the type:

 m
       X4.20.2010 X4.19.2010   X4.16.2010
 [1,]  0.008319468 0. -0.008250825
 [2,]  0.005574136 0.01816118  0.073081608
 [3,] -0.047830688 0.01612903 -0.030239833
 [4,]           NA         NA           NA
 [5,]  0.008746356 0.02848576 -0.025566107
 [6,] -0.007990868 0. -0.02667

 I want to get the sum of each column. Normally I would do:

 apply(m,2,sum)

 but I get:

 apply(m,2,sum)
 X4.20.2010 X4.19.2010 X4.16.2010
        NA         NA         NA

 This is because of the presence of NA in m. How do you the equivalent of
 sum(m[1:6,1],na.rm=TRUE)
 using apply?

 Many thanks!
 --
 View this message in context: 
 http://r.789695.n4.nabble.com/tapply-function-with-NA-tp2164930p2164930.html
 Sent from the R help mailing list archive at Nabble.com.

 __
 R-help@r-project.org mailing list
 https://stat.ethz.ch/mailman/listinfo/r-help
 PLEASE do read the posting guide http://www.R-project.org/posting-guide.html
 and provide commented, minimal, self-contained, reproducible code.


__
R-help@r-project.org mailing list
https://stat.ethz.ch/mailman/listinfo/r-help
PLEASE do read the posting guide http://www.R-project.org/posting-guide.html
and provide commented, minimal, self-contained, reproducible code.


[R] [R-pkgs] hash-2.0.0

2010-05-10 Thread Christopher Brown
The hash-2.0.0 has been released to CRAN.

This package implements a data structure similar to hashes in Perl and
dictionaries in Python but with a purposefully R flavor. For objects of
appreciable size, access using hashes outperforms native named lists and
vectors.

This version accounts for changes in R-2.11.0 and is optimized for both
speed and usabiity.

Some references:

http://opendatagroup.com/2010/04/26/hash-2-0-0/
http://opendatagroup.com/2009/07/26/hash-package-for-r/


Basic Examples:

  h - hash( keys=letters, values=1:26 )
  h - hash( letters, 1:26 )

  h$a # 1
  h$foo - bar

  h[ foo ]
  h[[ foo ]]

  clear(h)
  rm(h)


Warmest Regards,

Chris

Christopher Brown
Principal
Open Data
 http://www.opendatagroup.com
 http://blog.opendatagroup.com

[[alternative HTML version deleted]]

___
R-packages mailing list
r-packa...@r-project.org
https://stat.ethz.ch/mailman/listinfo/r-packages

__
R-help@r-project.org mailing list
https://stat.ethz.ch/mailman/listinfo/r-help
PLEASE do read the posting guide http://www.R-project.org/posting-guide.html
and provide commented, minimal, self-contained, reproducible code.


Re: [R] scope of a function + lazy evaluation

2010-05-10 Thread Patrick Burns

You are missing 'force'.

See 'The R Inferno' page 90.

In this case you can define:

f - function(y) { force(y); function() y}


On 10/05/2010 11:06, sayan dasgupta wrote:

Hey guys,

I have a doubt here , It is something simple I guess, what am I missing out
here ??


f- function(y) function() y
tmp- vector(list, 5)
for (i in 1:5) tmp[[i]]- f(i)
tmp[[1]]() # returns 5;

z- f(6)
tmp[[1]]() # still returns 5; it should return 6 ideally right ???

Even if  I dont evaluate the function tmp[[1]] before i.e I do
rm(list=ls())
f- function(y) function() y
tmp- vector(list, 5)
for (i in 1:5) tmp[[i]]- f(i)
  z- f(6)
tmp[[1]]() # it still returns 5; it should return 6 ideally right ???

[[alternative HTML version deleted]]

__
R-help@r-project.org mailing list
https://stat.ethz.ch/mailman/listinfo/r-help
PLEASE do read the posting guide http://www.R-project.org/posting-guide.html
and provide commented, minimal, self-contained, reproducible code.



--
Patrick Burns
pbu...@pburns.seanet.com
http://www.burns-stat.com
(home of 'Some hints for the R beginner'
and 'The R Inferno')

__
R-help@r-project.org mailing list
https://stat.ethz.ch/mailman/listinfo/r-help
PLEASE do read the posting guide http://www.R-project.org/posting-guide.html
and provide commented, minimal, self-contained, reproducible code.


[R] [R-pkgs] Bayesian change point package bcp 2.2.0 available

2010-05-10 Thread Jay Emerson
Version 2.2.0 of package bcp is now available.  It replaces the suggests of
NetWorkSpaces (previously used for optional parallel MCMC) with the
dependency on package foreach, giving greater flexibility and supporting a
wider range of parallel backends (see doSNOW, doMC, etc...).

For those unfamiliar with foreach (thanks to Steve Weston for this
contribution), it's a beautiful and highly portable looping construct which
can run sequentially or in parallel based on the user's actions (rather than
the programmer's choices).  We think other package authors might want to
consider taking advantage of it for tasks that might be computationally
intensive and could be easily done in parallel.  Some vignettes are
available at http://cran.r-project.org/web/packages/foreach/index.html.

Jay Emerson  Chandra Erdman

-- 
John W. Emerson (Jay)
Associate Professor of Statistics
Department of Statistics
Yale University
http://www.stat.yale.edu/~jay

[[alternative HTML version deleted]]

___
R-packages mailing list
r-packa...@r-project.org
https://stat.ethz.ch/mailman/listinfo/r-packages

__
R-help@r-project.org mailing list
https://stat.ethz.ch/mailman/listinfo/r-help
PLEASE do read the posting guide http://www.R-project.org/posting-guide.html
and provide commented, minimal, self-contained, reproducible code.


Re: [R] scope of a function + lazy evaluation

2010-05-10 Thread sayan dasgupta
Hey
thanks for your help ,
But thats not exactly the problem I have
See I am fine  with
tmp[[1]]() being = 5 and not 1; but then

for (i in 1:5) tmp[[i]] - f(i)
 z - f(6)
tmp[[1]]() ## should  give 6 right ? Because f(6) was last evaluate so in
parent.frame() y should be 6 ???


On Mon, May 10, 2010 at 3:44 PM, Patrick Burns pbu...@pburns.seanet.comwrote:

 You are missing 'force'.

 See 'The R Inferno' page 90.

 In this case you can define:

 f - function(y) { force(y); function() y}



 On 10/05/2010 11:06, sayan dasgupta wrote:

 Hey guys,

 I have a doubt here , It is something simple I guess, what am I missing
 out
 here ??


 f- function(y) function() y
 tmp- vector(list, 5)
 for (i in 1:5) tmp[[i]]- f(i)
 tmp[[1]]() # returns 5;

 z- f(6)
 tmp[[1]]() # still returns 5; it should return 6 ideally right ???

 Even if  I dont evaluate the function tmp[[1]] before i.e I do
 rm(list=ls())
 f- function(y) function() y
 tmp- vector(list, 5)
 for (i in 1:5) tmp[[i]]- f(i)
  z- f(6)
 tmp[[1]]() # it still returns 5; it should return 6 ideally right ???

[[alternative HTML version deleted]]

 __
 R-help@r-project.org mailing list
 https://stat.ethz.ch/mailman/listinfo/r-help
 PLEASE do read the posting guide
 http://www.R-project.org/posting-guide.html
 and provide commented, minimal, self-contained, reproducible code.


 --
 Patrick Burns
 pbu...@pburns.seanet.com
 http://www.burns-stat.com
 (home of 'Some hints for the R beginner'
 and 'The R Inferno')


[[alternative HTML version deleted]]

__
R-help@r-project.org mailing list
https://stat.ethz.ch/mailman/listinfo/r-help
PLEASE do read the posting guide http://www.R-project.org/posting-guide.html
and provide commented, minimal, self-contained, reproducible code.


Re: [R] scope of a function + lazy evaluation

2010-05-10 Thread Duncan Murdoch

sayan dasgupta wrote:

Hey guys,

I have a doubt here , It is something simple I guess, what am I missing out
here ??


f - function(y) function() y
tmp - vector(list, 5)
for (i in 1:5) tmp[[i]] - f(i)
tmp[[1]]() # returns 5;

z - f(6)
tmp[[1]]() # still returns 5; it should return 6 ideally right ???
  


No, each time you call f you create a new y variable in its local 
evaluation frame.  So all 6 of your y variables are different.  However, 
the first 5 of them are all defined by the same expression, i.e. i.  
Thus the first time they are evaluated they will each get the current 
value of that variable.  After the first evaluation, the value will be 
fixed, because that is when the value of y is forced.  So for example,


 f - function(y) function() y
 tmp - vector(list, 5)
 for (i in 1:5) tmp[[i]] - f(i)
 tmp[[1]]() # returns 5;
[1] 5

 i - 10
 tmp[[2]]()
[1] 10
 tmp[[1]]()
[1] 5


Even if  I dont evaluate the function tmp[[1]] before i.e I do
rm(list=ls())
f - function(y) function() y
tmp - vector(list, 5)
for (i in 1:5) tmp[[i]] - f(i)
 z - f(6)
tmp[[1]]() # it still returns 5; it should return 6 ideally right ???
  


See above.

Duncan Murdoch

[[alternative HTML version deleted]]

__
R-help@r-project.org mailing list
https://stat.ethz.ch/mailman/listinfo/r-help
PLEASE do read the posting guide http://www.R-project.org/posting-guide.html
and provide commented, minimal, self-contained, reproducible code.



__
R-help@r-project.org mailing list
https://stat.ethz.ch/mailman/listinfo/r-help
PLEASE do read the posting guide http://www.R-project.org/posting-guide.html
and provide commented, minimal, self-contained, reproducible code.


Re: [R] scope of a function + lazy evaluation

2010-05-10 Thread Gabor Grothendieck
When you call a function R passes a promise to it for each argument.
 A promise consists of the unevaluated variable together with the
environment in which it should evaluate the variable when time comes
to evaluate it.  Thus tmp[[1]] contains function(y) y and in the
environment of function(y) y there is a promise y to evaluate i the
first time that y is actually used.  When you write tmp[[1]]() the
promise is evaluated and since i is 5 at the point that is what you
get.  If f had actually used y when it was called then you would have
gotten 1.

On Mon, May 10, 2010 at 6:06 AM, sayan dasgupta kitt...@gmail.com wrote:
 Hey guys,

 I have a doubt here , It is something simple I guess, what am I missing out
 here ??


 f - function(y) function() y
 tmp - vector(list, 5)
 for (i in 1:5) tmp[[i]] - f(i)
 tmp[[1]]() # returns 5;

 z - f(6)
 tmp[[1]]() # still returns 5; it should return 6 ideally right ???

 Even if  I dont evaluate the function tmp[[1]] before i.e I do
 rm(list=ls())
 f - function(y) function() y
 tmp - vector(list, 5)
 for (i in 1:5) tmp[[i]] - f(i)
  z - f(6)
 tmp[[1]]() # it still returns 5; it should return 6 ideally right ???

        [[alternative HTML version deleted]]

 __
 R-help@r-project.org mailing list
 https://stat.ethz.ch/mailman/listinfo/r-help
 PLEASE do read the posting guide http://www.R-project.org/posting-guide.html
 and provide commented, minimal, self-contained, reproducible code.


__
R-help@r-project.org mailing list
https://stat.ethz.ch/mailman/listinfo/r-help
PLEASE do read the posting guide http://www.R-project.org/posting-guide.html
and provide commented, minimal, self-contained, reproducible code.


[R] Problems executing cor(dataset) function in R 2.11.0 for OS X ( It works fine in R 2.10.1)

2010-05-10 Thread Ruben Garcia Berasategui
 Dear all,
when trying to replicate John M. Quick's example for correlations between 
multiple variables posted on:

http://rtutorialseries.blogspot.com/2009/11/r-tutorial-series-zero-order.html

with R 2.11.0 (GUI 1.33) using my MacBook Pro with OX X 10.5.8 I got the 
following error message


 datavar-read.csv(dataset_readingTests.csv)
 cor(datavar)
Error in cor(datavar) : 'x' must be numeric

The funny thing is that when I tried to do the same example using R 2.10.1, it 
worked fine.
Any ideas regarding how to solve this problem?
Many thanks in advance,
Ruben

__
R-help@r-project.org mailing list
https://stat.ethz.ch/mailman/listinfo/r-help
PLEASE do read the posting guide http://www.R-project.org/posting-guide.html
and provide commented, minimal, self-contained, reproducible code.


Re: [R] Problems executing cor(dataset) function in R 2.11.0 for OS X ( It works fine in R 2.10.1)

2010-05-10 Thread Duncan Murdoch

On 10/05/2010 7:36 AM, Ruben Garcia Berasategui wrote:

 Dear all,
when trying to replicate John M. Quick's example for correlations between 
multiple variables posted on:

http://rtutorialseries.blogspot.com/2009/11/r-tutorial-series-zero-order.html

with R 2.11.0 (GUI 1.33) using my MacBook Pro with OX X 10.5.8 I got the 
following error message


  

datavar-read.csv(dataset_readingTests.csv)
cor(datavar)


Error in cor(datavar) : 'x' must be numeric

The funny thing is that when I tried to do the same example using R 2.10.1, it 
worked fine.
Any ideas regarding how to solve this problem?


I would think the first step would be to ask Mr. Quick what's wrong.

Duncan Murdoch

__
R-help@r-project.org mailing list
https://stat.ethz.ch/mailman/listinfo/r-help
PLEASE do read the posting guide http://www.R-project.org/posting-guide.html
and provide commented, minimal, self-contained, reproducible code.


[R] Rd files must have a non-empty \title

2010-05-10 Thread Assa Yeroslaviz
Hello everybody,

I'm trying to install a package I have built. This package contains three
scripts with various functions (S3 as well as S4 classes)

I run at first the package.skeleton command with:

 package.skeleton(affyAnalysis, namespace=TRUE, code_files =c(defS3.R,
defS4.R, qc.R))

affyAnalysis is the name of the supposed package and the three script are
listed at the end.
results:
Creating directories ...
Creating DESCRIPTION ...
Creating NAMESPACE ...
Creating Read-and-delete-me ...
Copying code files ...
Making help files ...
Done.
Further steps are described in './affyAnalysis/Read-and-delete-me'.
afterwards i tried to install the made package but i encountered a problem
with the RD files.

As I am working on windows I did it with the R CMD Install -l
path/to/library affyAnalysis.
As a result I'm getting the error message:
C:\Dokumente und Einstellungen\balt\DesktopR CMD INSTALL affyAnalysis
* installing to library 'C:\Programme\R\R-2.11.0\library'
* installing *source* package 'affyAnalysis' ...
** R
** preparing package for lazy loading
** help
Warning: ./man/affyAnalysis-package.Rd:35: All text must be in a section
Warning: ./man/affyAnalysis-package.Rd:36: All text must be in a section
Warning: ./man/z_-methods.Rd: \name should not contain !, | or @
*** installing help indices
Error in Rd_info(db[[i]]) : Rd files must have a non-empty \title.
See chapter 'Writing R documentation' in manual 'Writing R Extensions'.
* removing 'C:\Programme\R\R-2.11.0\library/affyAnalysis'

My question is is there a way around it?
I don't want to publish this package. It's just a compendium of various
functions I often use, so i wrote them as a package.
How can I install this package under R-2.11.0 without so much trouble as
changing ALL of my RD files?

THX for the help

Assa

 R.version
platform   i386-pc-mingw32
arch   i386
os mingw32
system i386, mingw32
status
major  2
minor  11.0
year   2010
month  04
day22
svn rev51801
language   R
version.string R version 2.11.0 (2010-04-22)

[[alternative HTML version deleted]]

__
R-help@r-project.org mailing list
https://stat.ethz.ch/mailman/listinfo/r-help
PLEASE do read the posting guide http://www.R-project.org/posting-guide.html
and provide commented, minimal, self-contained, reproducible code.


Re: [R] randomForest: sampling with replacement?

2010-05-10 Thread Liaw, Andy
See the replace argument in ?randomForest.

Andy 

 -Original Message-
 From: r-help-boun...@r-project.org 
 [mailto:r-help-boun...@r-project.org] On Behalf Of Dimitri 
 Liakhovitski
 Sent: Friday, May 07, 2010 12:21 PM
 To: r-help
 Subject: [R] randomForest: sampling with replacement?
 
 Hello!
 
 I know that as trees are constructed, the root node contains 
 a bootstrap sample of data of the same size as original the data set.
 Would you please confirm that it is a bootstrap sample with 
 replacement?
 I think it is - just wanted to make sure.
 Thank you very much!
 
 --
 Dimitri Liakhovitski
 Ninah.com
 
 __
 R-help@r-project.org mailing list
 https://stat.ethz.ch/mailman/listinfo/r-help
 PLEASE do read the posting guide 
 http://www.R-project.org/posting-guide.html
 and provide commented, minimal, self-contained, reproducible code.
 
Notice:  This e-mail message, together with any attachme...{{dropped:11}}

__
R-help@r-project.org mailing list
https://stat.ethz.ch/mailman/listinfo/r-help
PLEASE do read the posting guide http://www.R-project.org/posting-guide.html
and provide commented, minimal, self-contained, reproducible code.


Re: [R] Machine Learning and R

2010-05-10 Thread Liaw, Andy
I've not seen the book myself, but Graham Williams (author of the rattle
package) has been working on a book that perhaps may fit your need.

http://datamining.togaware.com/survivor/index.html

Andy

From: Wensui Liu
 
 good question!
 if there is such a book, i'd also like to read as well.
 
 On Sun, May 9, 2010 at 7:41 AM, Ralf B ralf.bie...@gmail.com wrote:
  Hi all,
 
  I am looking for a good book that covers Machine Learning 
 as a whole 
  and provides examples in R while not over focusing on the 
 math (such 
  as in 'Elements of Statistical Learning') but rather on 
 descriptions 
  and examples. I am relatively new to R and ML and, while solving 
  problems with R, I want to learn the main concepts, techniques and 
  problem categories. Can anybody here recommend good books? Does 
  anybody know a site that lists good books about R?
 
  Ralf
 
  __
  R-help@r-project.org mailing list
  https://stat.ethz.ch/mailman/listinfo/r-help
  PLEASE do read the posting guide 
  http://www.R-project.org/posting-guide.html
  and provide commented, minimal, self-contained, reproducible code.
 
 
 
 
 --
 ==
 WenSui Liu
 wens...@paypal.com
 statcompute.spaces.live.com
 
 __
 R-help@r-project.org mailing list
 https://stat.ethz.ch/mailman/listinfo/r-help
 PLEASE do read the posting guide 
 http://www.R-project.org/posting-guide.html
 and provide commented, minimal, self-contained, reproducible code.
 
Notice:  This e-mail message, together with any attachme...{{dropped:11}}

__
R-help@r-project.org mailing list
https://stat.ethz.ch/mailman/listinfo/r-help
PLEASE do read the posting guide http://www.R-project.org/posting-guide.html
and provide commented, minimal, self-contained, reproducible code.


[R] Installing randomForest on Ubuntu Errors

2010-05-10 Thread Steve_Friedman

Hello,

I've tried to install randomForest on a Ubuntu 8.04 Hardy Heron system.

I've repeatedly rec'd the error:

 install.packages(randomForest, dependencies = TRUE)

ERROR:  compiliation failed for package 'randomForest'
** Removing '/home/admuser/R/i486-pc-linux-gnu-library/2.6/randomForest'

The downloaded packages are in
   /tmp/RtmpkDsmTK/downloaded_packages
Warning message:
In install.packages(randomForest, dependencies = TRUE) :
  installation of package 'randomForest' had a non-zero exit status



The package loads correctly on my windows box, but not on the linux side.

Is this package available for Ubuntu Linux, if so what should I do to
install it?

Thanks

Steve


Steve Friedman Ph. D.
Spatial Statistical Analyst
Everglades and Dry Tortugas National Park
950 N Krome Ave (3rd Floor)
Homestead, Florida 33034

steve_fried...@nps.gov
Office (305) 224 - 4282
Fax (305) 224 - 4147

__
R-help@r-project.org mailing list
https://stat.ethz.ch/mailman/listinfo/r-help
PLEASE do read the posting guide http://www.R-project.org/posting-guide.html
and provide commented, minimal, self-contained, reproducible code.


Re: [R] Installing randomForest on Ubuntu Errors

2010-05-10 Thread Liaw, Andy
I don't know much about how permissions are managed under ubuntu, but
you can try a couple of things:

- Dirk had worked very hard at automating building of CRAN packages for
Debian (from which Ubuntu is based).  I believe randomForest is among
one of those available.  I'm not sure if you need to set up special
respository, but the idea is that you should be able to install R
packages via apt-get.

- Try installing in your home directory and see if that works:  Make a
directory under your home directory, e.g., called Rlibs.  Then in R,
do install.packages(randomForest, lib.loc=~/Rlibs, depend=TRUE).  If
that works, then your original problem is likely caused by permission.

Andy

From: steve_fried...@nps.gov
 
 Hello,
 
 I've tried to install randomForest on a Ubuntu 8.04 Hardy 
 Heron system.
 
 I've repeatedly rec'd the error:
 
  install.packages(randomForest, dependencies = TRUE)
 
 ERROR:  compiliation failed for package 'randomForest'
 ** Removing 
 '/home/admuser/R/i486-pc-linux-gnu-library/2.6/randomForest'
 
 The downloaded packages are in
/tmp/RtmpkDsmTK/downloaded_packages
 Warning message:
 In install.packages(randomForest, dependencies = TRUE) :
   installation of package 'randomForest' had a non-zero exit status
 
 
 
 The package loads correctly on my windows box, but not on the 
 linux side.
 
 Is this package available for Ubuntu Linux, if so what should 
 I do to install it?
 
 Thanks
 
 Steve
 
 
 Steve Friedman Ph. D.
 Spatial Statistical Analyst
 Everglades and Dry Tortugas National Park 950 N Krome Ave 
 (3rd Floor) Homestead, Florida 33034
 
 steve_fried...@nps.gov
 Office (305) 224 - 4282
 Fax (305) 224 - 4147
 
 __
 R-help@r-project.org mailing list
 https://stat.ethz.ch/mailman/listinfo/r-help
 PLEASE do read the posting guide 
 http://www.R-project.org/posting-guide.html
 and provide commented, minimal, self-contained, reproducible code.
 
Notice:  This e-mail message, together with any attachme...{{dropped:11}}

__
R-help@r-project.org mailing list
https://stat.ethz.ch/mailman/listinfo/r-help
PLEASE do read the posting guide http://www.R-project.org/posting-guide.html
and provide commented, minimal, self-contained, reproducible code.


Re: [R] Rd files must have a non-empty \title

2010-05-10 Thread Duncan Murdoch

On 10/05/2010 7:43 AM, Assa Yeroslaviz wrote:

Hello everybody,

I'm trying to install a package I have built. This package contains three
scripts with various functions (S3 as well as S4 classes)

I run at first the package.skeleton command with:

 package.skeleton(affyAnalysis, namespace=TRUE, code_files =c(defS3.R,
defS4.R, qc.R))

affyAnalysis is the name of the supposed package and the three script are
listed at the end.
results:
Creating directories ...
Creating DESCRIPTION ...
Creating NAMESPACE ...
Creating Read-and-delete-me ...
Copying code files ...
Making help files ...
Done.
Further steps are described in './affyAnalysis/Read-and-delete-me'.
afterwards i tried to install the made package but i encountered a problem
with the RD files.

As I am working on windows I did it with the R CMD Install -l
path/to/library affyAnalysis.
As a result I'm getting the error message:
C:\Dokumente und Einstellungen\balt\DesktopR CMD INSTALL affyAnalysis
* installing to library 'C:\Programme\R\R-2.11.0\library'
* installing *source* package 'affyAnalysis' ...
** R
** preparing package for lazy loading
** help
Warning: ./man/affyAnalysis-package.Rd:35: All text must be in a section
Warning: ./man/affyAnalysis-package.Rd:36: All text must be in a section
Warning: ./man/z_-methods.Rd: \name should not contain !, | or @
*** installing help indices
Error in Rd_info(db[[i]]) : Rd files must have a non-empty \title.
See chapter 'Writing R documentation' in manual 'Writing R Extensions'.
* removing 'C:\Programme\R\R-2.11.0\library/affyAnalysis'

My question is is there a way around it?
I don't want to publish this package. It's just a compendium of various
functions I often use, so i wrote them as a package.
How can I install this package under R-2.11.0 without so much trouble as
changing ALL of my RD files?
  


You could delete them all (or move them to another directory).  You 
won't pass R CMD check, but it sounds as though you don't care about that.


Alternatively, you could write a small function to replace the comment 
asking for a title with a junk title.  Just loop over all the Rd files, 
read them, substitute for the default title line, and write them back.  
If you don't want informative help files, it's really not much work to 
make uninformative ones.


Duncan Murdoch


THX for the help

Assa

 R.version
platform   i386-pc-mingw32
arch   i386
os mingw32
system i386, mingw32
status
major  2
minor  11.0
year   2010
month  04
day22
svn rev51801
language   R
version.string R version 2.11.0 (2010-04-22)

[[alternative HTML version deleted]]

__
R-help@r-project.org mailing list
https://stat.ethz.ch/mailman/listinfo/r-help
PLEASE do read the posting guide http://www.R-project.org/posting-guide.html
and provide commented, minimal, self-contained, reproducible code.



__
R-help@r-project.org mailing list
https://stat.ethz.ch/mailman/listinfo/r-help
PLEASE do read the posting guide http://www.R-project.org/posting-guide.html
and provide commented, minimal, self-contained, reproducible code.


Re: [R] randomForest: sampling with replacement?

2010-05-10 Thread Max Kuhn
 Would you please confirm that it is a bootstrap sample with
 replacement?

Someone should note that the definition of a bootstrap sample is a
sample with replacement (usually of size n).

I've read quite a few papers where they claim to be using the
bootstrap. Upon further review (sometimes to the code) they often are
doing permutation testing-type randomization. This is mostly in
biological journals.

There are also at least paper that describe the out of box random
forests error rate...

Max

__
R-help@r-project.org mailing list
https://stat.ethz.ch/mailman/listinfo/r-help
PLEASE do read the posting guide http://www.R-project.org/posting-guide.html
and provide commented, minimal, self-contained, reproducible code.


Re: [R] lattice: customising panel.segments using groups

2010-05-10 Thread Felix Andrews
On 9 May 2010 15:25, Ky Mathews ky.math...@sydney.edu.au wrote:
 Hi,



 I want to customise the segments on an xyplot. Below is a simple example
 of what I'm trying to do...



 #Example dataset

  x - c(-0.25, 0.25, 0.8)

  y - c(-0.5, 0, 0.75)

  gp - c(A, I, C)

  my.data - cbind.data.frame(x,y,gp)



 #setting up the parameters to customise the lines with.

 ltype - c(1,2,3)

 env.col - c(red, black, blue)

 env.lwd - c(1.25, 0.75, 1.25)



 # Lattice plot

 xyplot(y ~ x,

             data= my.data,

             groups = gp,

             type = l,

   panel= panel.superpose,

   panel.groups = function(x,y, subscripts=subscripts,
 groups=groups,...){

      panel.segments(0, 0, x[groups], y[groups], lty = ltype[groups],
 lwd=env.lwd[groups], col = env.col[groups])})


The 'panel.groups' function is passed a special argument
'group.number', not 'groups'... (Also you do not need to subset 'x'
and 'y' because they are already subsetted by 'panel.superpose'.)

xyplot(y ~ x, my.data,
groups = gp, type = l,
panel = panel.superpose,
panel.groups = function(x, y, group.number, ...) {
panel.segments(0, 0, x, y, lty = ltype[group.number],
lwd=env.lwd[group.number], col = env.col[group.number])})


...however, panel.superpose does this for you automatically, so you
only need to pass 'lty', 'lwd' and 'col' arguments to xyplot, and
these will be split up by group.number and passed on as single values.
 For an even better solution, it is recommended to use par.settings
(or trellis.par.set for global settings), so that 'auto.key' will
work:

xyplot(y ~ x, my.data,
groups = gp, type = l,
panel = panel.superpose,
panel.groups = function(x, y, ...)
panel.xyplot(c(0, x), c(0, y), ...),
par.settings = simpleTheme(col = c(red, black, blue),
lwd = c(1.25, 0.75, 1.25), lty = c(1,2,3)),
auto.key = list(lines = TRUE, points = FALSE))


HTH
-Felix





 #The problem:
 I don't seem to have this quite right, as the resulting plot seems to
 ignore the lty, lwd, col that I set up.

 I tried to do this by using trellis.par.set/get but simply got confused.



 If I remove lty, lwd and col specifications I simply get all segments
 with the same attributes. i.e. it seems to be ignoring the groups





 #The answer I want:

 Segment1 has co-ordinates (0,0, x2=-0.25, y2 = -0.5) and be solid, red
 and of width 1.25

 Segment2 has co-ordinates (0,0, x2=0.25, y2 = 0) and be dashed, black
 and of width 0.75

 Segment3 has co-ordinates (0,0, x2=0.8, y2 = 0.75) and be dotted, blue
 and of width 1.25



 Any help is much appreciated.



 Thanks and regards,

 Ky Mathews




        [[alternative HTML version deleted]]

 __
 R-help@r-project.org mailing list
 https://stat.ethz.ch/mailman/listinfo/r-help
 PLEASE do read the posting guide http://www.R-project.org/posting-guide.html
 and provide commented, minimal, self-contained, reproducible code.




-- 
Felix Andrews / 安福立
Postdoctoral Fellow
Integrated Catchment Assessment and Management (iCAM) Centre
Fenner School of Environment and Society [Bldg 48a]
The Australian National University
Canberra ACT 0200 Australia
M: +61 410 400 963
T: + 61 2 6125 4670
E: felix.andr...@anu.edu.au
CRICOS Provider No. 00120C
-- 
http://www.neurofractal.org/felix/

__
R-help@r-project.org mailing list
https://stat.ethz.ch/mailman/listinfo/r-help
PLEASE do read the posting guide http://www.R-project.org/posting-guide.html
and provide commented, minimal, self-contained, reproducible code.


Re: [R] How to estimate whether overfitting?

2010-05-10 Thread Frank E Harrell Jr

On 05/10/2010 12:32 AM, bbslover wrote:


many thanks .  I can try to use test set with 100 samples.

anther question is that how can I rationally split my data to training set
and test set? (training set with 108 samples, and test set with 100 samples)

as I  know, the test set should the same distribute to the training set. and
what method can deal with it to rationally split?

and what packages in R can deal with splitting training/test set rationally
question?


if the split is random. it seems to need many times splits, and the average
results consider as the final results.

however, I want to several methods to perform split and get the firm
training set and test set instead of random split.

training set and test set should like this:ideally, the division must be
performed sunch that points representing both traing and training set are
distributed within the hole feature space occupied by the entire dataset,
and each point of the test set is close to at least one point of the
training set. this approach ensures that the similarity principle can be
enmployed for the output prediction of the test set. Certainly,this
condition can not always be satistied.

thus, generally, what algorithms often be perform to split? and more
rational? some paper often say, they split the data set  randomly, thus,
what is randomly?  just selection random? or have some clear method? e.g.
output order,  I really know, which package can do with split data
rationally?

other, if one want to get the better results, some tips can be done. e.g.
they can select test set again and again, and use the test set with best
results as final test set and say that the test set was selectd randomly,
but it is not true random, it is false.

thank you, sorry to so many questions. but it puzzled me always.  up to now,
I have no good method to split rationally my data into training set and test
set.

at last, split training and test set should be done before modeling, and it
seems that this can be done just from featrue? (som)  ( or feature and
output?(alogorithm spxy. paper:a method for calibration and validation
subset partioning)  or just output?(output order)).

but always, often there are many features to be calculated. and some featrue
is zero or low standard deviation(sd0.5),  should we delete these features
before split the whole data?

and use the remaining feature to split data, and just using the training set
to build the regression model and to perform feature selection as well as to
do cross-validation,  and the independent test set just used to test the
built model, yes?

maybe, my thinking is not clear about the whole model precess. but I think
it is like this:
1) get samples
2) calculate features
3) preprocess features calculated (e.g.remove zero)
4)rational split data into training and test set (always puzzle me, how to
split on earth?)
5)build model and at the same time tune parameter of model  based on the
resample methods using just training set. and get the final model.
6) test the model performance using independent test set (unseen samples).
7) estimate the model. good? or bad?  overfitting?  (generally, what case is
overfitting? can you give me a example? as i know, it is overfitting when
the trainging set fit good, but the independent test set is bad,but what is
good ? what is bad?r2=0.94 in the training set and r2=0.70 in the test,
in this case, the model is overfitting?  the model can be accepted?  and
generally what model can be well accetpt?)
8) conclusion. how is the model.

above is my thinking.  and many question wait for answering.

thanks

kevin





Kevin: I'm sorry I don't have time to deal with such a long note, but 
briefly data splitting is not a good idea no matter how you do it unless 
N  perhaps 20,000.  I suggest resampling, e.g., either the bootstrap 
with 300 resamples or 50-fold repeats of 10-fold cross-validation. 
Among other places these are implemented in my rms package.


Frank

--
Frank E Harrell Jr   Professor and ChairmanSchool of Medicine
 Department of Biostatistics   Vanderbilt University

__
R-help@r-project.org mailing list
https://stat.ethz.ch/mailman/listinfo/r-help
PLEASE do read the posting guide http://www.R-project.org/posting-guide.html
and provide commented, minimal, self-contained, reproducible code.


Re: [R] Installing randomForest on Ubuntu Errors

2010-05-10 Thread Dirk Eddelbuettel

Steve,

The list r-sig-debian is a more appropriate forum for this question as it is
dedicated to R on Debian / Ubuntu.

On 10 May 2010 at 08:24, steve_fried...@nps.gov wrote:
| 
| Hello,
| 
| I've tried to install randomForest on a Ubuntu 8.04 Hardy Heron system.
| 
| I've repeatedly rec'd the error:
| 
|  install.packages(randomForest, dependencies = TRUE)
| 
| ERROR:  compiliation failed for package 'randomForest'
| ** Removing '/home/admuser/R/i486-pc-linux-gnu-library/2.6/randomForest'

Can you produce the actual error?  Ie why did install.packages fail?

What happens when you down the source .tar.gz and try

sudo R CMD INSTALL randomForest_*.tar.gz 

instead?

Many of us happily run and install numerous packages from CRAN on Debian
and/or Ubuntu and I am sure we can help out, but it would be best if you came
over to r-sig-debian (and you need to subscribe first).

Hth, Dirk

 
| The downloaded packages are in
|/tmp/RtmpkDsmTK/downloaded_packages
| Warning message:
| In install.packages(randomForest, dependencies = TRUE) :
|   installation of package 'randomForest' had a non-zero exit status
| 
| 
| 
| The package loads correctly on my windows box, but not on the linux side.
| 
| Is this package available for Ubuntu Linux, if so what should I do to
| install it?
| 
| Thanks
| 
| Steve
| 
| 
| Steve Friedman Ph. D.
| Spatial Statistical Analyst
| Everglades and Dry Tortugas National Park
| 950 N Krome Ave (3rd Floor)
| Homestead, Florida 33034
| 
| steve_fried...@nps.gov
| Office (305) 224 - 4282
| Fax (305) 224 - 4147
| 
| __
| R-help@r-project.org mailing list
| https://stat.ethz.ch/mailman/listinfo/r-help
| PLEASE do read the posting guide http://www.R-project.org/posting-guide.html
| and provide commented, minimal, self-contained, reproducible code.

-- 
  Regards, Dirk

__
R-help@r-project.org mailing list
https://stat.ethz.ch/mailman/listinfo/r-help
PLEASE do read the posting guide http://www.R-project.org/posting-guide.html
and provide commented, minimal, self-contained, reproducible code.


[R] Comparing Survival: Mantel-Byar Test

2010-05-10 Thread Raphael Fraser
The Mantel-Byar test is a simple modification to the logrank test. Can
R perform this test or any other similar test?

Raphael

__
R-help@r-project.org mailing list
https://stat.ethz.ch/mailman/listinfo/r-help
PLEASE do read the posting guide http://www.R-project.org/posting-guide.html
and provide commented, minimal, self-contained, reproducible code.


[R] dot plot with error bands

2010-05-10 Thread Alexey Bessudnov

Dear all,

I'm trying to create a dot plot with error bands with

Dotplot(labels ~ Cbind(estimate, lower, upper), data=For.plot)

where estimate, lower and upper are numerical vectors, and labels is a 
character vector that contains labels.


The problem is that labels are automatically sorted alphabetically, and 
I want them to be sorted by estimate (as in my data frame). This should 
be straightforward, but unfortunately being new to R I can't figure out 
how to do this. I'll appreciate your guidance.


Best,

Alexey

__
R-help@r-project.org mailing list
https://stat.ethz.ch/mailman/listinfo/r-help
PLEASE do read the posting guide http://www.R-project.org/posting-guide.html
and provide commented, minimal, self-contained, reproducible code.


[R] smoothing parameter in locfit package

2010-05-10 Thread FMH
Hi, 

In the locfit package, could someone please let me know the automatic selection 
of smoothing parameter if Gauss kernel density function is used as weight 
function?

thanks
Fir


  
[[alternative HTML version deleted]]

__
R-help@r-project.org mailing list
https://stat.ethz.ch/mailman/listinfo/r-help
PLEASE do read the posting guide http://www.R-project.org/posting-guide.html
and provide commented, minimal, self-contained, reproducible code.


[R] Extracting a subset

2010-05-10 Thread Silvano

Hi,

I have a dataset with many variables and observations.
The variable Group has two levels: C and P,
the Month variable has four levels: 0, 1, 2 and 3.

I want to extract a subset of the variable Weight, 
considering only 1 and 3 levels for Months of the Group 
variable.


I tried the command subset but it did not work. Any 
suggestions?


Thanks,

--
Silvano Cesar da Costa
Departamento de Estatística
Universidade Estadual de Londrina
Fone: 3371-4346

__
R-help@r-project.org mailing list
https://stat.ethz.ch/mailman/listinfo/r-help
PLEASE do read the posting guide http://www.R-project.org/posting-guide.html
and provide commented, minimal, self-contained, reproducible code.


Re: [R] update R 2.11.0,there is error when usi ng plot(), how can I do?

2010-05-10 Thread bbslover

now. it is ok.  I uninstall R2.11.0, then delete an packages in  the library,
and install again R2.11.0. ok, it does works.  

thank you!
-- 
View this message in context: 
http://r.789695.n4.nabble.com/update-R-2-11-0-there-is-error-when-using-plot-how-can-I-do-tp2164517p2165235.html
Sent from the R help mailing list archive at Nabble.com.

__
R-help@r-project.org mailing list
https://stat.ethz.ch/mailman/listinfo/r-help
PLEASE do read the posting guide http://www.R-project.org/posting-guide.html
and provide commented, minimal, self-contained, reproducible code.


[R] empty matrix

2010-05-10 Thread anderson nuel
Dear r-help,

Could you help me to find the function which create an empty matrix.

I use matrix(), but it gives *a single value that is NA  and  length of this
matrix is 1.*
**
*Best Regards*

[[alternative HTML version deleted]]

__
R-help@r-project.org mailing list
https://stat.ethz.ch/mailman/listinfo/r-help
PLEASE do read the posting guide http://www.R-project.org/posting-guide.html
and provide commented, minimal, self-contained, reproducible code.


[R] dot plot with error bands

2010-05-10 Thread Alexey Bessudnov

Dear all,

I'm trying to create a dot plot with error bands with

Dotplot(labels ~ Cbind(estimate, lower, upper), data=For.plot)

where estimate, lower and upper are numerical vectors, and labels is a 
character vector that contains labels.


The problem is that labels are automatically sorted alphabetically, and 
I want them to be sorted by estimate (as in my data frame). This should 
be straightforward, but unfortunately being new to R I can't figure out 
how to do this. I'll appreciate your guidance.


Best,

Alexey

__
R-help@r-project.org mailing list
https://stat.ethz.ch/mailman/listinfo/r-help
PLEASE do read the posting guide http://www.R-project.org/posting-guide.html
and provide commented, minimal, self-contained, reproducible code.


[R] estimating mean income

2010-05-10 Thread Erwin Kalvelagen
Hi:

This is more a statistical question than an R question (apologies!).

I have some income data as follows:

$5000 : 598
$5000-$1 : 2586

$65001-$7 : 202
$70001+ : 446

I.e an open ended income class for incomes  $70k.  
What would be the best way to estimate mean income? 
Something like using a Pareto distribution?. 

Any suggestion or references are much appreciated.

Thanks very much.

Erwin

__
R-help@r-project.org mailing list
https://stat.ethz.ch/mailman/listinfo/r-help
PLEASE do read the posting guide http://www.R-project.org/posting-guide.html
and provide commented, minimal, self-contained, reproducible code.


Re: [R] Extracting a subset

2010-05-10 Thread Uwe Ligges



On 10.05.2010 14:35, Silvano wrote:

Hi,

I have a dataset with many variables and observations.
The variable Group has two levels: C and P,
the Month variable has four levels: 0, 1, 2 and 3.

I want to extract a subset of the variable Weight, considering only 1
and 3 levels for Months of the Group variable.

I tried the command subset but it did not work. Any suggestions?



Tell us how you used it and how your data looks like and we may be in a 
position to help afterwards,.


 PLEASE do read the posting guide
 http://www.R-project.org/posting-guide.html
 and provide commented, minimal, self-contained, reproducible code.


Uwe Ligges



Thanks,

--
Silvano Cesar da Costa
Departamento de Estatística
Universidade Estadual de Londrina
Fone: 3371-4346

__
R-help@r-project.org mailing list
https://stat.ethz.ch/mailman/listinfo/r-help
PLEASE do read the posting guide
http://www.R-project.org/posting-guide.html
and provide commented, minimal, self-contained, reproducible code.


__
R-help@r-project.org mailing list
https://stat.ethz.ch/mailman/listinfo/r-help
PLEASE do read the posting guide http://www.R-project.org/posting-guide.html
and provide commented, minimal, self-contained, reproducible code.


Re: [R] empty matrix

2010-05-10 Thread Uwe Ligges



On 10.05.2010 13:15, anderson nuel wrote:

Dear r-help,

Could you help me to find the function which create an empty matrix.


Help is:

?matrix

Uwe Ligges




I use matrix(), but it gives *a single value that is NA  and  length of this
matrix is 1.*
**
*Best Regards*

[[alternative HTML version deleted]]

__
R-help@r-project.org mailing list
https://stat.ethz.ch/mailman/listinfo/r-help
PLEASE do read the posting guide http://www.R-project.org/posting-guide.html
and provide commented, minimal, self-contained, reproducible code.


__
R-help@r-project.org mailing list
https://stat.ethz.ch/mailman/listinfo/r-help
PLEASE do read the posting guide http://www.R-project.org/posting-guide.html
and provide commented, minimal, self-contained, reproducible code.


Re: [R] Extracting a subset

2010-05-10 Thread Ivan Calandra

Hi,
My first suggestion would be to supply a sample data (maybe using the 
function dput) showing what you have, what you want to do, and what 
you've tried (you say that subset() didn't work but we don't know how 
you've typed it).

Then, we'll see!
Ivan



Le 5/10/2010 14:35, Silvano a écrit :

Hi,

I have a dataset with many variables and observations.
The variable Group has two levels: C and P,
the Month variable has four levels: 0, 1, 2 and 3.

I want to extract a subset of the variable Weight, considering only 1 
and 3 levels for Months of the Group variable.


I tried the command subset but it did not work. Any suggestions?

Thanks,

--
Silvano Cesar da Costa
Departamento de Estatística
Universidade Estadual de Londrina
Fone: 3371-4346

__
R-help@r-project.org mailing list
https://stat.ethz.ch/mailman/listinfo/r-help
PLEASE do read the posting guide 
http://www.R-project.org/posting-guide.html

and provide commented, minimal, self-contained, reproducible code.



--
Ivan CALANDRA
PhD Student
University of Hamburg
Biozentrum Grindel und Zoologisches Museum
Abt. Säugetiere
Martin-Luther-King-Platz 3
D-20146 Hamburg, GERMANY
+49(0)40 42838 6231
ivan.calan...@uni-hamburg.de

**
http://www.for771.uni-bonn.de
http://webapp5.rrz.uni-hamburg.de/mammals/eng/mitarbeiter.php

__
R-help@r-project.org mailing list
https://stat.ethz.ch/mailman/listinfo/r-help
PLEASE do read the posting guide http://www.R-project.org/posting-guide.html
and provide commented, minimal, self-contained, reproducible code.


Re: [R] empty matrix

2010-05-10 Thread Duncan Murdoch

On 10/05/2010 7:15 AM, anderson nuel wrote:

Dear r-help,

Could you help me to find the function which create an empty matrix.

I use matrix(), but it gives *a single value that is NA  and  length of this
matrix is 1.*



Not sure what you mean by an empty matrix, but here's one interpretation:

matrix(numeric(0), 0,0)

This gives a 0 by 0 matrix.

Duncan Murdoch

__
R-help@r-project.org mailing list
https://stat.ethz.ch/mailman/listinfo/r-help
PLEASE do read the posting guide http://www.R-project.org/posting-guide.html
and provide commented, minimal, self-contained, reproducible code.


Re: [R] empty matrix

2010-05-10 Thread Ista Zahn
I think matrix(nrow=0, ncol=0) will do it.

-Ista

On Mon, May 10, 2010 at 7:15 AM, anderson nuel anderson@gmail.comwrote:

 Dear r-help,

 Could you help me to find the function which create an empty matrix.

 I use matrix(), but it gives *a single value that is NA  and  length of
 this
 matrix is 1.*
 **
 *Best Regards*

[[alternative HTML version deleted]]

 __
 R-help@r-project.org mailing list
 https://stat.ethz.ch/mailman/listinfo/r-help
 PLEASE do read the posting guide
 http://www.R-project.org/posting-guide.html
 and provide commented, minimal, self-contained, reproducible code.




-- 
Ista Zahn
Graduate student
University of Rochester
Department of Clinical and Social Psychology
http://yourpsyche.org

[[alternative HTML version deleted]]

__
R-help@r-project.org mailing list
https://stat.ethz.ch/mailman/listinfo/r-help
PLEASE do read the posting guide http://www.R-project.org/posting-guide.html
and provide commented, minimal, self-contained, reproducible code.


Re: [R] dot plot with error bands

2010-05-10 Thread David Winsemius


On May 10, 2010, at 10:00 AM, Alexey Bessudnov wrote:


Dear all,

I'm trying to create a dot plot with error bands with

Dotplot(labels ~ Cbind(estimate, lower, upper), data=For.plot)

where estimate, lower and upper are numerical vectors, and labels is  
a character vector that contains labels.


The problem is that labels are automatically sorted alphabetically,  
and I want them to be sorted by estimate (as in my data frame). This  
should be straightforward, but unfortunately being new to R I can't  
figure out how to do this. I'll appreciate your guidance.


Have you tried making labels (an unfortunate choice for a variable  
name, BTW) a factor variable with levels in the order of your desire?


(Also being new to R, you may not recognize the difference between a  
factor variable and a character vector, so producing a more explicit  
description of the dataframe For.plot with the str function ought to  
be your next contribution to this thread if the above solution is not  
effective.


--
David Winsemius, MD
West Hartford, CT

__
R-help@r-project.org mailing list
https://stat.ethz.ch/mailman/listinfo/r-help
PLEASE do read the posting guide http://www.R-project.org/posting-guide.html
and provide commented, minimal, self-contained, reproducible code.


Re: [R] smoothing parameter in locfit package

2010-05-10 Thread Liaw, Andy
See the kern argument in ?locfit.raw.

Andy 

From: FMH
 Hi, 
 
 In the locfit package, could someone please let me know the 
 automatic selection of smoothing parameter if Gauss kernel 
 density function is used as weight function?
 
 thanks
 Fir
 
 
   
   [[alternative HTML version deleted]]
 
 __
 R-help@r-project.org mailing list
 https://stat.ethz.ch/mailman/listinfo/r-help
 PLEASE do read the posting guide 
 http://www.R-project.org/posting-guide.html
 and provide commented, minimal, self-contained, reproducible code.
 
Notice:  This e-mail message, together with any attachme...{{dropped:11}}

__
R-help@r-project.org mailing list
https://stat.ethz.ch/mailman/listinfo/r-help
PLEASE do read the posting guide http://www.R-project.org/posting-guide.html
and provide commented, minimal, self-contained, reproducible code.


Re: [R] Extracting a subset

2010-05-10 Thread Silvano

Hi,

I used

#--
# Análise do PESO -
#--
pesom1 = cbind(PESO[MÊS==1])
pesom3 = cbind(PESO[MÊS==3])

#- Grupo Placebo -
pesom1p = pesom1[GRUPO=='P']
pesom3p = pesom3[GRUPO=='P']
t.test(pesom1p, pesom3p, paired=T)

#- Grupo Cogumelo -
pesom1c = pesom1[GRUPO=='C']
pesom3c = pesom3[GRUPO=='C']
t.test(pesom1c, pesom3c, paired=T)

to compare PESO between MÊS (Months) inside GRUPO 
(t-paired). CICLO was fixed.



But, I have two CICLOS, and I would like compare PESO 
between MÊS==3 (CICLO 3) and MÊS==6 (CICLO 6) inside GRUPO 
(1 and 2).


My file has (part of data):

Ciclo   Mês Grupo   Peso
3   0   1   62.2
3   0   1   67.0
3   0   1   71.5
3   0   2   70.0
3   0   2   71.0
3   0   2   69.5
3   1   1   60.0
3   1   1   51.2
3   1   1   70.0
3   1   2   59.0
3   1   2   56.5
3   1   2   70.0
3   2   1   68.0
3   2   1   69.0
3   2   1   66.0
3   2   2   56.0
3   2   2   53.0
3   2   2   96.0
3   3   1   67.0
3   3   1   70.0
3   3   1   67.0
3   3   2   71.5
3   3   2   70.0
3   3   2   71.0
6   0   1   66.2
6   0   1   65.0
6   0   1   73.5
6   0   2   78.0
6   0   2   71.0
6   0   2   62.5
6   1   1   63.0
6   1   1   51.2
6   1   1   72.0
6   1   2   59.0
6   1   2   57.5
6   1   2   79.0
6   2   1   78.0
6   2   1   66.0
6   2   1   63.0
6   2   2   52.0
6   2   2   54.0
6   2   2   93.0
6   3   1   62.0
6   3   1   71.0
6   3   1   63.0
6   3   2   78.5
6   3   2   75.0
6   3   2   74.0
6   4   1   63.2
6   4   1   62.0
6   4   1   73.5
6   4   2   73.0
6   4   2   72.0
6   4   2   65.5
6   5   1   67.0
6   5   1   57.2
6   5   1   70.0
6   5   2   59.0
6   5   2   56.5
6   5   2   71.0
6   6   1   61.0
6   6   1   61.0
6   6   1   61.0
6   6   2   52.0
6   6   2   53.0
6   6   2   96.0

--
Silvano Cesar da Costa
Departamento de Estatística
Universidade Estadual de Londrina
Fone: 3371-4346
--
- Original Message - 
From: Ivan Calandra ivan.calan...@uni-hamburg.de

To: r-help@r-project.org
Sent: Monday, May 10, 2010 11:26 AM
Subject: Re: [R] Extracting a subset


Hi,
My first suggestion would be to supply a sample data (maybe 
using the
function dput) showing what you have, what you want to do, 
and what
you've tried (you say that subset() didn't work but we don't 
know how

you've typed it).
Then, we'll see!
Ivan



Le 5/10/2010 14:35, Silvano a écrit :

Hi,

I have a dataset with many variables and observations.
The variable Group has two levels: C and P,
the Month variable has four levels: 0, 1, 2 and 3.

I want to extract a subset of the variable Weight, 
considering only 1 and 3 levels for Months of the Group 
variable.


I tried the command subset but it did not work. Any 
suggestions?


Thanks,

--
Silvano Cesar da Costa
Departamento de Estatística
Universidade Estadual de Londrina
Fone: 3371-4346

__
R-help@r-project.org mailing list
https://stat.ethz.ch/mailman/listinfo/r-help
PLEASE do read the posting guide 
http://www.R-project.org/posting-guide.html
and provide commented, minimal, self-contained, 
reproducible code.




--
Ivan CALANDRA
PhD Student
University of Hamburg
Biozentrum Grindel und Zoologisches Museum
Abt. Säugetiere
Martin-Luther-King-Platz 3
D-20146 Hamburg, GERMANY
+49(0)40 42838 6231
ivan.calan...@uni-hamburg.de

**
http://www.for771.uni-bonn.de
http://webapp5.rrz.uni-hamburg.de/mammals/eng/mitarbeiter.php

__
R-help@r-project.org mailing list
https://stat.ethz.ch/mailman/listinfo/r-help
PLEASE do read the posting guide 
http://www.R-project.org/posting-guide.html
and provide commented, minimal, self-contained, reproducible 
code.


__
R-help@r-project.org mailing list
https://stat.ethz.ch/mailman/listinfo/r-help
PLEASE do read the posting guide http://www.R-project.org/posting-guide.html
and provide commented, minimal, self-contained, reproducible code.


[R] Plotting residuals from a sem object

2010-05-10 Thread Jesse Young
R experts - 

I'm using John Fox's sem package to analyze a simple path model (two correlated 
predictor variables directly influencing a single criterion variable):

Predictor1 - Criterion
Predictor2 - Criterion
Predictor1 - Predictor2

I'm giving a presentation on this material next week, and I'd like to use 
component-residual plots (i.e., partial residual plots) to help the audience 
visualize the effect of one the predictor variables on the criterion, 
independent of the other predictor variable.  In a simple multiple regression, 
I'd use the cr.plots function from Fox's CAR package for this, or just plot 
residuals from two models regressing my predictor of interest and the criterion 
variable separately on the other predictor variable.  Is there a way to produce 
similar plots for a path model (or generate the appropriate residuals to just 
do the plot myself)?

thanks,
Jesse

_
Jesse W. Young, Ph.D.
Assistant Professor
Department of Anatomy and Neurobiology
Northeastern Ohio Universities College
of Medicine (NEOUCOM)

Office phone: (330) 325-6304
Lab phone: (330) 325-6307
Fax: (330) 325-5916
Email: jwyo...@neoucom.edu
Web page: www.jessewyoung.com

__
R-help@r-project.org mailing list
https://stat.ethz.ch/mailman/listinfo/r-help
PLEASE do read the posting guide http://www.R-project.org/posting-guide.html
and provide commented, minimal, self-contained, reproducible code.


Re: [R] Plotting log-axis with the exponential base to a plot with the default logarithm base 10

2010-05-10 Thread Xianwen Chen
Dear Elisabeth,

I'm not sure if I have understood your question -- are you trying
to use a different logarithmic base?

 Kind regards,

 Xianwen

On Sun, May 9, 2010 at 8:05 PM, Elisabeth Bjerke Rastad
ebr...@post.uit.no wrote:
 Hello!

 I have a problem which I have tried to solve for several days now..

 I have plottet a lineplot.CI in the library sciplot, and I am trying to
 plot it with a logaritmic y-axis (with exponential base).

 The problem is that; when I type log y, the axis transforms into the
 logaritmic of base 10.

 I wonder if someeone could tell me how to specify that I would like to use
 the exponential logaritmic y-axis. I have tried a lot (but obviously not
 all, I guess this problem is possible to solve..)

 Hope you would like to help me! Thank you a lot in advance!!

 Greetings,

 Elisabeth B. Råstad
 (Master's student, Norway)

 __
 R-help@r-project.org mailing list
 https://stat.ethz.ch/mailman/listinfo/r-help
 PLEASE do read the posting guide http://www.R-project.org/posting-guide.html
 and provide commented, minimal, self-contained, reproducible code.




-- 
Xianwen Chen, M.Sc.
Scientific assistant, BFE, UiT (www.uit.no)
Tel.: +47 776 46 112 | Fax: +47 776 46 020

__
R-help@r-project.org mailing list
https://stat.ethz.ch/mailman/listinfo/r-help
PLEASE do read the posting guide http://www.R-project.org/posting-guide.html
and provide commented, minimal, self-contained, reproducible code.


Re: [R] How to estimate whether overfitting?

2010-05-10 Thread Bert Gunter
(Near) non-identifiability (especially in nonlinear models, which include
linear mixed effects models, Bayesian hierarchical models, etc.) is
typically a strong clue; usually indicated by software complaints (e.g.
convergence failures, running up against iteration limits, etc.). 

However this is sufficient-ish, not necessary: over-fitting frequently
occurs even without such overt complaints. It should also be said that,
except for identifiability,  over-fitting is not a well-defined
statistical term: it depends on the scientific context.


Bert Gunter
Genentech Nonclinical Biostatistics
 
 -Original Message-
From: r-help-boun...@r-project.org [mailto:r-help-boun...@r-project.org] On
Behalf Of Steve Lianoglou
Sent: Sunday, May 09, 2010 6:13 PM
To: David Winsemius
Cc: r-help@r-project.org; bbslover
Subject: Re: [R] How to estimate whether overfitting?

On Sun, May 9, 2010 at 11:53 AM, David Winsemius dwinsem...@comcast.net
wrote:

 On May 9, 2010, at 9:20 AM, bbslover wrote:


 1. is there some criterion to estimate overfitting?  e.g. R2 and Q2 in
the
 training set, as well as R2 in the test set, when means overfitting.  
for
 example,  in my data, I have R2=0.94 for the training set and  for the
 test
 set R2=0.70, is overfitting?
 2. in this scatter, can one say this overfitting?

 3. my result is obtained by svm, and the sample are 156 and 52 for the
 training and test sets, and predictors are 96,   In this case, can svm be
 employed to perform prediction?   whether the number of the predictors
are
 too many ?


 I think you need to buy a copy of Hastie, Tibshirani, and Friedman and do
 some self-study of chapters 7 and 12.

And you don't even have to buy it before you can start studying since
the PDF is available here:
http://www-stat.stanford.edu/~tibs/ElemStatLearn/

Having a hard cover is always handy, tho ..
-steve

-- 
Steve Lianoglou
Graduate Student: Computational Systems Biology
 | Memorial Sloan-Kettering Cancer Center
 | Weill Medical College of Cornell University
Contact Info: http://cbio.mskcc.org/~lianos/contact

__
R-help@r-project.org mailing list
https://stat.ethz.ch/mailman/listinfo/r-help
PLEASE do read the posting guide http://www.R-project.org/posting-guide.html
and provide commented, minimal, self-contained, reproducible code.

__
R-help@r-project.org mailing list
https://stat.ethz.ch/mailman/listinfo/r-help
PLEASE do read the posting guide http://www.R-project.org/posting-guide.html
and provide commented, minimal, self-contained, reproducible code.


Re: [R] post-hoc t-tests, for lme / lmer models

2010-05-10 Thread Kay Cichini

no one?
any pointers would really be greatly appreciated!

thanks,
kay


-

Kay Cichini
Postgraduate student
Institute of Botany
Univ. of Innsbruck


-- 
View this message in context: 
http://r.789695.n4.nabble.com/post-hoc-t-tests-for-lme-lmer-models-tp2133959p2171820.html
Sent from the R help mailing list archive at Nabble.com.

__
R-help@r-project.org mailing list
https://stat.ethz.ch/mailman/listinfo/r-help
PLEASE do read the posting guide http://www.R-project.org/posting-guide.html
and provide commented, minimal, self-contained, reproducible code.


Re: [R] smoothing parameter in locfit package

2010-05-10 Thread Liaw, Andy
Loader's book is the ultimate reference (other than the source code...).  I 
don't have it with me right now, so can't tell you for sure.  However, it seems 
like an odd thing to do to use a different kernel at the selection step than 
the final estimation step.  I should think if you specify the kernel, it is 
used regardless of how the bandwidth is chosen.

Andy 

From: FMH 
 
 Thank you Andy. I'm aware of that but my question is about 
 the way  the locfit package use to automatically determine 
 the bandwidth/smoothing parameter if Gauss kernel density 
 function is used as the weight function. Does the generalized 
 cross validation criteria is imposed here or does the value 
 of the bandwidth is fixed (by default) in the package, unless 
 we specify our own value?
 
 
 Cheers,
 Fir
 
 
 
 - Original Message 
 From: Liaw, Andy andy_l...@merck.com
 To: FMH kagba2...@yahoo.com; r-help@r-project.org
 Sent: Mon, May 10, 2010 4:00:51 PM
 Subject: RE: [R] smoothing parameter in locfit package
 
 See the kern argument in ?locfit.raw.
 
 Andy 
 
 From: FMH
  Hi,
  
  In the locfit package, could someone please let me know the 
 automatic 
  selection of smoothing parameter if Gauss kernel density 
 function is 
  used as weight function?
  
  thanks
  Fir
  
  
       
      [[alternative HTML version deleted]]
  
  __
  R-help@r-project.org mailing list
  https://stat.ethz.ch/mailman/listinfo/r-help
  PLEASE do read the posting guide
  http://www.R-project.org/posting-guide.html
  and provide commented, minimal, self-contained, reproducible code.
  
 Notice:  This e-mail message, together with any attachments, 
 contains information of Merck  Co., Inc. (One Merck Drive, 
 Whitehouse Station, New Jersey, USA 08889), and/or its 
 affiliates Direct contact information for affiliates is available at
 http://www.merck.com/contact/contacts.html) that may be 
 confidential, proprietary copyrighted and/or legally 
 privileged. It is intended solely for the use of the 
 individual or entity named on this message. If you are not 
 the intended recipient, and have received this message in 
 error, please notify us immediately by reply e-mail and then 
 delete it from your system.
 
 
   
 
Notice:  This e-mail message, together with any attachme...{{dropped:11}}

__
R-help@r-project.org mailing list
https://stat.ethz.ch/mailman/listinfo/r-help
PLEASE do read the posting guide http://www.R-project.org/posting-guide.html
and provide commented, minimal, self-contained, reproducible code.


[R] [Fwd: Re: Plotting log-axis with the exponential base to a plot with the default logarithm base 10]

2010-05-10 Thread Elisabeth Bjerke Rastad


Hello!
Thank you for answering!
What I am trying to do is to plot my raw values (biomass of different
species) on a logaritmic y-axis with the base of e. When I type log=y,
the axis transforms into a logaritmic axis with the base of 10.

Best regards,
Elisabeth

 Dear Elisabeth,

 I'm not sure if I have understood your question -- are you trying
 to use a different logarithmic base?

  Kind regards,

  Xianwen

 On Sun, May 9, 2010 at 8:05 PM, Elisabeth Bjerke Rastad
 ebr...@post.uit.no wrote:
 Hello!

 I have a problem which I have tried to solve for several days now..

 I have plottet a lineplot.CI in the library sciplot, and I am trying
 to
 plot it with a logaritmic y-axis (with exponential base).

 The problem is that; when I type log y, the axis transforms into the
 logaritmic of base 10.

 I wonder if someeone could tell me how to specify that I would like to
 use
 the exponential logaritmic y-axis. I have tried a lot (but obviously not
 all, I guess this problem is possible to solve..)

 Hope you would like to help me! Thank you a lot in advance!!

 Greetings,

 Elisabeth B. RÃ¥stad
 (Master's student, Norway)

 __
 R-help@r-project.org mailing list
 https://stat.ethz.ch/mailman/listinfo/r-help
 PLEASE do read the posting guide
 http://www.R-project.org/posting-guide.html
 and provide commented, minimal, self-contained, reproducible code.




 --
 Xianwen Chen, M.Sc.
 Scientific assistant, BFE, UiT (www.uit.no)
 Tel.: +47 776 46 112 | Fax: +47 776 46 020



__
R-help@r-project.org mailing list
https://stat.ethz.ch/mailman/listinfo/r-help
PLEASE do read the posting guide http://www.R-project.org/posting-guide.html
and provide commented, minimal, self-contained, reproducible code.


Re: [R] Random walk

2010-05-10 Thread Albyn Jones
Sums of correlated increments have the same correlation as the original
variables...

 library(mvtnorm)
 X- matrix(0,nrow=1000,ncol=2)
 for(i in 1:1000){
 Y - rmvnorm(1000,mean=mu,sigma=S)
 X[i,] - apply(Y,2,sum)
 }
 cor(Y)
  [,1]  [,2]
[1,] 1.000 0.4909281
[2,] 0.4909281 1.000

So, unless you meant that you want the _sample_ correlation to be
pre-specified, you are all set.

albyn

On Sun, May 09, 2010 at 09:20:25PM -0400, Sergio Andrés Estay Cabrera wrote:
 Hi everybody,


 I am trying to generate two random walks with an specific correlation, for 
 example, two random walks of 200 time steps with a correlation 0.7.

 I built the random walks with:

 x-cumsum(rnorm(200, mean=0,sd=1))
 y-cumsum(rnorm(200, mean=0,sd=1))

 but I don't know how to fix the correlation between them.

 With white noise is easy to fix the correlation using the function rmvnorm 
 in the package mvtnorm

 I surfed in the web in the searchable mail archives in the R web site but 
 no references appears.

 If you have some advices to solve this problems I would be very thankful.

 Thanks in advance.

 Sergio A. Estay
 *CASEB *
 Departamento de Ecología
 Universidad Catolica de Chile

 -- 
 “La disciplina no tiene ningún mérito en circunstancias ideales. ” – Habor 
 Mallow

 __
 R-help@r-project.org mailing list
 https://stat.ethz.ch/mailman/listinfo/r-help
 PLEASE do read the posting guide http://www.R-project.org/posting-guide.html
 and provide commented, minimal, self-contained, reproducible code.


__
R-help@r-project.org mailing list
https://stat.ethz.ch/mailman/listinfo/r-help
PLEASE do read the posting guide http://www.R-project.org/posting-guide.html
and provide commented, minimal, self-contained, reproducible code.


[R] ggplot2 and geom_pointrange

2010-05-10 Thread michael westphal
Hello:

I am using the ggplot2 package on R 2.10.1.  I am plotting points using 
geom_pointrange.  Is there a way to overlay hashmarks on the points, 
specifically the median and the min and max of the range? 

Cheers,

Michael


  
[[alternative HTML version deleted]]

__
R-help@r-project.org mailing list
https://stat.ethz.ch/mailman/listinfo/r-help
PLEASE do read the posting guide http://www.R-project.org/posting-guide.html
and provide commented, minimal, self-contained, reproducible code.


Re: [R] smoothing parameter in locfit package

2010-05-10 Thread FMH
Thank you Andy. I'm aware of that but my question is about the way  the locfit 
package use to automatically determine the bandwidth/smoothing parameter if 
Gauss kernel density function is used as the weight function. Does the 
generalized cross validation criteria is imposed here or does the value of the 
bandwidth is fixed (by default) in the package, unless we specify our own value?


Cheers,
Fir



- Original Message 
From: Liaw, Andy andy_l...@merck.com
To: FMH kagba2...@yahoo.com; r-help@r-project.org
Sent: Mon, May 10, 2010 4:00:51 PM
Subject: RE: [R] smoothing parameter in locfit package

See the kern argument in ?locfit.raw.

Andy 

From: FMH
 Hi, 
 
 In the locfit package, could someone please let me know the 
 automatic selection of smoothing parameter if Gauss kernel 
 density function is used as weight function?
 
 thanks
 Fir
 
 
      
     [[alternative HTML version deleted]]
 
 __
 R-help@r-project.org mailing list
 https://stat.ethz.ch/mailman/listinfo/r-help
 PLEASE do read the posting guide 
 http://www.R-project.org/posting-guide.html
 and provide commented, minimal, self-contained, reproducible code.
 
Notice:  This e-mail message, together with any attachments, contains
information of Merck  Co., Inc. (One Merck Drive, Whitehouse Station,
New Jersey, USA 08889), and/or its affiliates Direct contact information
for affiliates is available at 
http://www.merck.com/contact/contacts.html) that may be confidential,
proprietary copyrighted and/or legally privileged. It is intended solely
for the use of the individual or entity named on this message. If you are
not the intended recipient, and have received this message in error,
please notify us immediately by reply e-mail and then delete it from 
your system.




__
R-help@r-project.org mailing list
https://stat.ethz.ch/mailman/listinfo/r-help
PLEASE do read the posting guide http://www.R-project.org/posting-guide.html
and provide commented, minimal, self-contained, reproducible code.


[R] Supercripting text

2010-05-10 Thread Shang Gao
Dear R users,

I recently developed a plotting function in R and introduced it to my 
coworkers. The function is designed to make plotting easier and more efficient, 
which will in turn be more cost-effective for the company. The reviews for the 
function have been positive thus far, except for one issue -- addition of 
superscripts to the title. We need superscipts in the titles sometimes to 
highlight footnotes which appear at the bottom of the plots.

The syntax for supersciprts, however, is rather cumbersome, especially in 
titles since it needs to be bolded. So far the only way of superscripting is to 
use the expression() function. But to go about formatting the text such that it 
appears bolded as a title in my plots, I would have to type in the command
 text(expression(bold(paste(text for title,^1))))

In some cases, the plot would require 3 footnotes to be shown, and the code 
would be
 text(expression(bold(paste(text for title,^1,  ^2,  ^3))).)

Most of my coworkers are still in the process of picking up R, some have never 
used R before. The above commands may be a little too much for them to handle.

Is there an easier way of superscripting texts in R? It would be great if any 
of you know of alternative ways other than using the expression() function.

I greatly appreciate your help.

Thank you.

Sincerely,
Shang


[[alternative HTML version deleted]]

__
R-help@r-project.org mailing list
https://stat.ethz.ch/mailman/listinfo/r-help
PLEASE do read the posting guide http://www.R-project.org/posting-guide.html
and provide commented, minimal, self-contained, reproducible code.


[R] System neutral Daylight Savings Time response?

2010-05-10 Thread Garrett Grolemund
I'm searching for an r command that will notify me if I create a time that
does not exist due to Daylight Savings Time. For example, if I run the
following command on a windows machine

 ISOdatetime(2010,03,14,2,10,0, tz = ) # My system time is set to the
United States Central Time Zone
[1] NA

R returns NA, which is the behavior I want. However, if I run the same
command on a mac, R returns a POSIXct object and I have to examine the
object manually to notice that I had tried to create an impossible time.

ISOdatetime(2010,03,14,2,10,0, tz = )
[1] 2010-03-14 01:10:00 CST

Is there a method of creating time objects in R that will always return NA
for non-existant times, no matter the operating system?

Thank you sincerely,
Garrett

[[alternative HTML version deleted]]

__
R-help@r-project.org mailing list
https://stat.ethz.ch/mailman/listinfo/r-help
PLEASE do read the posting guide http://www.R-project.org/posting-guide.html
and provide commented, minimal, self-contained, reproducible code.


Re: [R] Plotting log-axis with the exponential base to a plot with the default logarithm base 10

2010-05-10 Thread Xianwen Chen
Dear Elisabeth,

log(X) shall return logarithm of X, with the base of e.

If you are not sure, you can specify e to be the base, e.g.:
log(X,exp(1))
.

Snow in Tromsø is melting, so will your problem.

King regards,

Xianwen

On Mon, May 10, 2010 at 6:11 PM, Elisabeth Bjerke Rastad
ebr...@post.uit.no wrote:

 Hello!
 Thank you for answering!
 What I am trying to do is to plot my raw values (biomass of different
 species) on a logaritmic y-axis with the base of e. When I type log=y,
 the axis transforms into a logaritmic axis with the base of 10.

 Best regards,
 Elisabeth

  Dear Elisabeth,
 
      I'm not sure if I have understood your question -- are you trying
  to use a different logarithmic base?
 
                                                           Kind regards,
 
                                                           Xianwen
 
  On Sun, May 9, 2010 at 8:05 PM, Elisabeth Bjerke Rastad
  ebr...@post.uit.no wrote:
  Hello!
 
  I have a problem which I have tried to solve for several days now..
 
  I have plottet a lineplot.CI in the library sciplot, and I am trying
  to
  plot it with a logaritmic y-axis (with exponential base).
 
  The problem is that; when I type log y, the axis transforms into the
  logaritmic of base 10.
 
  I wonder if someeone could tell me how to specify that I would like to
  use
  the exponential logaritmic y-axis. I have tried a lot (but obviously not
  all, I guess this problem is possible to solve..)
 
  Hope you would like to help me! Thank you a lot in advance!!
 
  Greetings,
 
  Elisabeth B. Råstad
  (Master's student, Norway)
 
  __
  R-help@r-project.org mailing list
  https://stat.ethz.ch/mailman/listinfo/r-help
  PLEASE do read the posting guide
  http://www.R-project.org/posting-guide.html
  and provide commented, minimal, self-contained, reproducible code.
 
 
 
 
  --
  Xianwen Chen, M.Sc.
  Scientific assistant, BFE, UiT (www.uit.no)
  Tel.: +47 776 46 112 | Fax: +47 776 46 020
 
 





--
Xianwen Chen, M.Sc.
Scientific assistant, BFE, UiT (www.uit.no)
Tel.: +47 776 46 112 | Fax: +47 776 46 020

__
R-help@r-project.org mailing list
https://stat.ethz.ch/mailman/listinfo/r-help
PLEASE do read the posting guide http://www.R-project.org/posting-guide.html
and provide commented, minimal, self-contained, reproducible code.


Re: [R] Random walk

2010-05-10 Thread Sergio Andrés Estay Cabrera

Dear R users and specially Albyn and Giovanni,

thanks for your answers, but in fact I supposed the same at the 
beginning of my problem. However, when I generate the data seldom I 
obtain the expected correlation. For example using this code:


fz-function(n,t,rho){
f-NULL
for(i in 1:n){
s-rmvnorm(n=t,mean=c(0,0),sigma=matrix(c(1,rho,rho,1),ncol=2),method='svd')
paso-cor(cumsum(s[,1]),cumsum(s[,2]))
f-c(f,paso)}
f-f
}

and then plot the histogram of the results, it is possible to observe 
that the distribution of the values is asymmetric with most of the 
simulations close to 1 when the value of rho is higher than 0.3 and 
looks like a uniform distribution with values below 0.3.


I suspect than the only possibility is using a brute force algorithm.


Any advice would be helpful

Sergio A. Estay
*CASEB *
Departamento de Ecología
Universidad Catolica de Chile




Albyn Jones wrote:

Sums of correlated increments have the same correlation as the original
variables...

 library(mvtnorm)
 X- matrix(0,nrow=1000,ncol=2)
 for(i in 1:1000){
 Y - rmvnorm(1000,mean=mu,sigma=S)
 X[i,] - apply(Y,2,sum)
 }
 cor(Y)
  [,1]  [,2]
[1,] 1.000 0.4909281
[2,] 0.4909281 1.000

So, unless you meant that you want the _sample_ correlation to be
pre-specified, you are all set.

albyn

On Sun, May 09, 2010 at 09:20:25PM -0400, Sergio Andrés Estay Cabrera wrote:
  

Hi everybody,


I am trying to generate two random walks with an specific correlation, for 
example, two random walks of 200 time steps with a correlation 0.7.


I built the random walks with:

x-cumsum(rnorm(200, mean=0,sd=1))
y-cumsum(rnorm(200, mean=0,sd=1))

but I don't know how to fix the correlation between them.

With white noise is easy to fix the correlation using the function rmvnorm 
in the package mvtnorm


I surfed in the web in the searchable mail archives in the R web site but 
no references appears.


If you have some advices to solve this problems I would be very thankful.

Thanks in advance.

Sergio A. Estay
*CASEB *
Departamento de Ecología
Universidad Catolica de Chile

--
“La disciplina no tiene ningún mérito en circunstancias ideales. ” – Habor 
Mallow

__
R-help@r-project.org mailing list
https://stat.ethz.ch/mailman/listinfo/r-help
PLEASE do read the posting guide http://www.R-project.org/posting-guide.html
and provide commented, minimal, self-contained, reproducible code.





  



--
“La disciplina no tiene ningún mérito en circunstancias ideales. ” – Habor 
Mallow

__
R-help@r-project.org mailing list
https://stat.ethz.ch/mailman/listinfo/r-help
PLEASE do read the posting guide http://www.R-project.org/posting-guide.html
and provide commented, minimal, self-contained, reproducible code.


Re: [R] Random walk

2010-05-10 Thread David Winsemius


On May 10, 2010, at 2:55 PM, Sergio Andrés Estay Cabrera wrote:


Dear R users and specially Albyn and Giovanni,

thanks for your answers, but in fact I supposed the same at the  
beginning of my problem. However, when I generate the data seldom I  
obtain the expected correlation. For example using this code:


fz-function(n,t,rho){
f-NULL
for(i in 1:n){
s-rmvnorm(n=t,mean=c(0,0),sigma=matrix(c(1,rho,rho, 
1),ncol=2),method='svd')

paso-cor(cumsum(s[,1]),cumsum(s[,2]))
f-c(f,paso)}
f-f
}


I believe you may want to search on the term quantos model. If you  
have two independent Brownian processes W_1(t) and W_2(t) then W_3 =  
rho*W_1(t) +sqrt(1-rho^2)*W_2(t) will theoretically have correlation  
rho with W_1. Sampling will of course giving varying instantiation  
values.



--
David
citation: Financial Calculus, Baxter M; Rennie A (1996)


and then plot the histogram of the results, it is possible to  
observe that the distribution of the values is asymmetric with most  
of the simulations close to 1 when the value of rho is higher than  
0.3 and looks like a uniform distribution with values below 0.3.


I suspect than the only possibility is using a brute force algorithm.


Any advice would be helpful

Sergio A. Estay
*CASEB *
Departamento de Ecología
Universidad Catolica de Chile




Albyn Jones wrote:
Sums of correlated increments have the same correlation as the  
original

variables...

library(mvtnorm)
X- matrix(0,nrow=1000,ncol=2)
for(i in 1:1000){
Y - rmvnorm(1000,mean=mu,sigma=S)
X[i,] - apply(Y,2,sum)
}
cor(Y)
 [,1]  [,2]
[1,] 1.000 0.4909281
[2,] 0.4909281 1.000

So, unless you meant that you want the _sample_ correlation to be
pre-specified, you are all set.

albyn

On Sun, May 09, 2010 at 09:20:25PM -0400, Sergio Andrés Estay  
Cabrera wrote:



Hi everybody,


I am trying to generate two random walks with an specific  
correlation, for example, two random walks of 200 time steps with  
a correlation 0.7.


I built the random walks with:

x-cumsum(rnorm(200, mean=0,sd=1))
y-cumsum(rnorm(200, mean=0,sd=1))

but I don't know how to fix the correlation between them.

With white noise is easy to fix the correlation using the function  
rmvnorm in the package mvtnorm


I surfed in the web in the searchable mail archives in the R web  
site but no references appears.


If you have some advices to solve this problems I would be very  
thankful.


Thanks in advance.

Sergio A. Estay
*CASEB *
Departamento de Ecología
Universidad Catolica de Chile

--
“La disciplina no tiene ningún mérito en circunstancias ideales. ”  
– Habor Mallow


__
R-help@r-project.org mailing list
https://stat.ethz.ch/mailman/listinfo/r-help
PLEASE do read the posting guide http://www.R-project.org/posting-guide.html
and provide commented, minimal, self-contained, reproducible code.









--
“La disciplina no tiene ningún mérito en circunstancias ideales. ” –  
Habor Mallow


__
R-help@r-project.org mailing list
https://stat.ethz.ch/mailman/listinfo/r-help
PLEASE do read the posting guide http://www.R-project.org/posting-guide.html
and provide commented, minimal, self-contained, reproducible code.


David Winsemius, MD
West Hartford, CT

__
R-help@r-project.org mailing list
https://stat.ethz.ch/mailman/listinfo/r-help
PLEASE do read the posting guide http://www.R-project.org/posting-guide.html
and provide commented, minimal, self-contained, reproducible code.


Re: [R] Dimensions on svychisq on svydesign

2010-05-10 Thread Thomas Lumley



I think your problem is that HL07 is not a data frame but a list.  read.spss() 
produces a list rather than a data frame by default, and svydesign() requires a 
data frame.  You can use as.data.frame() to turn HL07 into a data.frame.

The weights also look a bit strange -- you have sampling probabilities ranging 
from 0.27 to 3 -- this isn't necessarily wrong but is unusual.

   -thomas

On Sun, 9 May 2010, Stefán Jónsson wrote:


Dear Forum

I a running  svychisq from the survey package and get errors with the number
of dimensions,
errors that I do not understand and do not know how to fix.

I ask you kindly to help me out.

The eror message follows with some information below. I hope there are
enough information to help me to fix the problem if not please let me know
what is needed

Best
Stefan Hrafn Jonsson


QHISQ2 - svychisq(~S73 + S36 , design = HL7design , statistic  = Chisq)

Error in design$variables[, as.character(rows)] :
 incorrect number of dimensions




HL07 - read.spss(C:/Users/Stefan/Desktop/export1.por, use.value.labels =
FALSE)

HL7design -svydesign(id=~ID2,strata=~STRATA, weights=~VIGT2, data=HL07 )

summary(HL7design)




summary(HL7design)

Stratified Independent Sampling design (with replacement)
svydesign(id = ~ID2, strata = ~STRATA, weights = ~VIGT2, data = HL07)
Probabilities:
  Min. 1st Qu.  MedianMean 3rd Qu.Max.
0.3779  0.6933  1.2880  1.3270  1.6970  3.0980
Stratum Sizes:
1   2   3   4   5   6  11  12  13  14  15  16
obs414 403 447 479 501 464 364 453 514 540 534 521
design.PSU 414 403 447 479 501 464 364 453 514 540 534 521
actual.PSU 414 403 447 479 501 464 364 453 514 540 534 521
Data variables:
NULL








xtabs(~HL07$S73 + HL07$S36 )
   HL07$S36
HL07$S73   1   2   3   4   5   6   7   8   9
  1 199 248 517 707 736 156  57  10   2
  2 458 294 636 992 543  55  18   3   3



QHISQ2 - svychisq(~HL07$S73 + HL07$S36 , design = HL7design , statistic

= Chisq)
Error in design$variables[, as.character(rows)] :
 incorrect number of dimensions




QHISQ2 - svychisq(~S73 + S36 , design = HL7design , statistic  = Chisq)

Error in design$variables[, as.character(rows)] :
 incorrect number of dimensions

[[alternative HTML version deleted]]

__
R-help@r-project.org mailing list
https://stat.ethz.ch/mailman/listinfo/r-help
PLEASE do read the posting guide http://www.R-project.org/posting-guide.html
and provide commented, minimal, self-contained, reproducible code.



Thomas Lumley   Assoc. Professor, Biostatistics
tlum...@u.washington.eduUniversity of Washington, Seattle
__
R-help@r-project.org mailing list
https://stat.ethz.ch/mailman/listinfo/r-help
PLEASE do read the posting guide http://www.R-project.org/posting-guide.html
and provide commented, minimal, self-contained, reproducible code.


[R] graphics::plot Organizing line types, line colors and generating matching legends...

2010-05-10 Thread Ralf B
Lets say I have a generated data frame with variables that follow a
naming convention:

title,a1,a2,b1,b2,b3,c1,c2,c3,c4...

I am plotting every column (starting from a1) as a line in a plot.
That works. However my diagram becomes very unorganized. Creating
legends is nice, but trying out different combinations requires me to
adjust my legend since it is generally disconnected from the data.

Is there an elegant way where R generates legends for its variables so
that the legend will fit the line and uses the column name as in the
legend? I guess I am asking for the basic Excel thing. I understand
that in the standard graphics package, this is not really intended.
Perhaps somebody can point me into a direction where this more easily
possible? Is it for example easier in gplot or lattice?

Ralf

__
R-help@r-project.org mailing list
https://stat.ethz.ch/mailman/listinfo/r-help
PLEASE do read the posting guide http://www.R-project.org/posting-guide.html
and provide commented, minimal, self-contained, reproducible code.


Re: [R] graphics::plot Organizing line types, line colors and generating matching legends...

2010-05-10 Thread baptiste auguie
Hi,

Lattice and ggplot2 are both ideally suited for this task. Consider
this example,

library(ggplot2)
d = data.frame(x=1:10, a1=rnorm(10), b1=rnorm(10))
m = melt(d, id =x) # reshape into long format

qplot(x, value, data=m, geom=path, colour=variable)

library(lattice)
xyplot(value~x, data=m, type=l, group=variable, auto.key=TRUE)


HTH,

baptiste

On 10 May 2010 21:29, Ralf B ralf.bie...@gmail.com wrote:
 Lets say I have a generated data frame with variables that follow a
 naming convention:

 title,a1,a2,b1,b2,b3,c1,c2,c3,c4...

 I am plotting every column (starting from a1) as a line in a plot.
 That works. However my diagram becomes very unorganized. Creating
 legends is nice, but trying out different combinations requires me to
 adjust my legend since it is generally disconnected from the data.

 Is there an elegant way where R generates legends for its variables so
 that the legend will fit the line and uses the column name as in the
 legend? I guess I am asking for the basic Excel thing. I understand
 that in the standard graphics package, this is not really intended.
 Perhaps somebody can point me into a direction where this more easily
 possible? Is it for example easier in gplot or lattice?

 Ralf

 __
 R-help@r-project.org mailing list
 https://stat.ethz.ch/mailman/listinfo/r-help
 PLEASE do read the posting guide http://www.R-project.org/posting-guide.html
 and provide commented, minimal, self-contained, reproducible code.


__
R-help@r-project.org mailing list
https://stat.ethz.ch/mailman/listinfo/r-help
PLEASE do read the posting guide http://www.R-project.org/posting-guide.html
and provide commented, minimal, self-contained, reproducible code.


[R] Bug in DEMA (Moving Average smoothing algoritm) ?

2010-05-10 Thread Ralf B
When running DEMA(data, 5) on a vector 'data' of length 5, my R engine
stops. Is this function or the R environment facing a bug here or am I
doing something wrong? DEMA should work if the smoothing window size
is the same size as the the data length, right?

(I am working with Eclipse 3.5. and the StatET environment.)

Ralf

__
R-help@r-project.org mailing list
https://stat.ethz.ch/mailman/listinfo/r-help
PLEASE do read the posting guide http://www.R-project.org/posting-guide.html
and provide commented, minimal, self-contained, reproducible code.


Re: [R] Supercripting text

2010-05-10 Thread Kevin Coombes
Sigh; that's because I forget to include it in the 'substitute' list.  
Here is a version that works (and has been tested...).


ourtitle - function(title, footnotes=NULL) {
 if (length(footnotes)  0) {
   fn - paste(footnotes, collapse=' ')
   title - eval(substitute(expression(bold(paste(title, ^fn))),
list(title=title, fn=fn)))
 }
 title
}

Shang Gao wrote:

I tried writing that function before, but the expression() command is not preserved when we 
incorporate it into a function(). The first example you gave me, if run in R, produced a title that 
reads title instead of stuff. It seems that in this case the expression() 
can't read the character string in the argument specified in the function() command.

-Original Message-
From: Kevin Coombes [mailto:kevin.r.coom...@gmail.com] 
Sent: Monday, May 10, 2010 12:08 PM

To: Shang Gao
Cc: r-help@r-project.org
Subject: Re: [R] Supercripting text

You probably want to write your own titling function to add 
superscripts.  The following code will serve as a reasonable starting point:


# make the title expression
ourtitle - function(title, footnotes=NULL) {
  if (length(footnotes)  0) {
fn - paste(footnotes, collapse=' ')
title - eval(substitute(expression(bold(paste(title, ^fn))), 
list(fn=fn)))

  }
  title
}
# example with footnotes
tt - ourtitle(stuff, 1:3)
plot(1, 1)
title(tt)
# example without footnotes
plot(1, 1)
title(ourtitle(stuff))

Note that the only way I could make this work was to combine the 
footnote list before calling the eval-substitute-expression code; I 
cannot get it to work by applying things ot a list.


Kevin

Shang Gao wrote:
  

Dear R users,

I recently developed a plotting function in R and introduced it to my 
coworkers. The function is designed to make plotting easier and more efficient, 
which will in turn be more cost-effective for the company. The reviews for the 
function have been positive thus far, except for one issue -- addition of 
superscripts to the title. We need superscipts in the titles sometimes to 
highlight footnotes which appear at the bottom of the plots.

The syntax for supersciprts, however, is rather cumbersome, especially in 
titles since it needs to be bolded. So far the only way of superscripting is to 
use the expression() function. But to go about formatting the text such that it 
appears bolded as a title in my plots, I would have to type in the command
  


text(expression(bold(paste(text for title,^1))))

  

In some cases, the plot would require 3 footnotes to be shown, and the code 
would be
  


text(expression(bold(paste(text for title,^1,  ^2,  ^3))).)

  

Most of my coworkers are still in the process of picking up R, some have never 
used R before. The above commands may be a little too much for them to handle.

Is there an easier way of superscripting texts in R? It would be great if any 
of you know of alternative ways other than using the expression() function.

I greatly appreciate your help.

Thank you.

Sincerely,
Shang


[[alternative HTML version deleted]]

__
R-help@r-project.org mailing list
https://stat.ethz.ch/mailman/listinfo/r-help
PLEASE do read the posting guide http://www.R-project.org/posting-guide.html
and provide commented, minimal, self-contained, reproducible code.
  



__
R-help@r-project.org mailing list
https://stat.ethz.ch/mailman/listinfo/r-help
PLEASE do read the posting guide http://www.R-project.org/posting-guide.html
and provide commented, minimal, self-contained, reproducible code.


Re: [R] Using R with screenreading software

2010-05-10 Thread Roopakshi Pathania

Hi,

I use R with a screen reader to do statistical and financial analysis. 

There are a couple of things that your student can do.

R was created on Linux, and then ported to Windows. So she can use R under 
Linux where the CLI is perfectly accessible.
There are 2 main screen readers for Linux: Speakup for the text console and 
Orca for the Gnome Desktop.
Also, ESS, an extension to Emacs can be used as well.

On Windows, the accessibility of R console can vary from screen reader to 
screen reader.
I mainly work on the R terminal found in C:\Program Files\R\R-2.10.0\bin. You 
could create a desktop shortcut for the Rterm.exe file.
If she is not comfortable typing directly on the R console, then a text editor 
can be used for the R code.
Any text editor can do, though I use Tinn-R, which is fairly accessible.
Tinn-R can be found here.
http://www.sciviews.org/Tinn-R/

The code can either be pasted in R or can be run directly by using the source 
function.
 source(C:/Test.r)
See ?source

R output can be sent to a text file using sink()

 # Open the connection
 sink(sink-Test.txt)

 # Enter something
 x - c(1, 2, 3, 4, 5)
 mean(x)

 # End the connection
 sink()

More helpful would be ?sink and ?capture.output

Depending on what she has to do, R can also be used through Excel which is 
again accessible.

Regards
Roopakshi from India
--
 
 Message: 29
 Date: Tue, 4 May 2010 15:41:41 +0200
 From: Rainer Scheuchenpflug
     scheuchenpf...@psychologie.uni-wuerzburg.de
 To: r-help@r-project.org
 Subject: [R] Using R with screenreading software
 Message-ID:
 002e01caeb8f$8736b2c0$95a418...@uni-wuerzburg.de
 Content-Type: text/plain;   
 charset=iso-8859-1
 
 Dear R-Experts,
 
 a student of mine tries to use the Windows-Rconsole with
 screen reading
 software (she is blind), and cannot access the command line
 (Menus are ok).
 The company which produces her screen reader tells her that
 this is due to
 the cursor used in Rconsole, which is static, not blinking.
 They maintain
 that if the cursor could be changed to a blinking one, she
 should be able to
 access the command line and outputs.
 
 For my last exam she used R in a Dosbox as workaround, but
 encountered other
 problems, esp. with scrolling. So: Is it possible to change
 the cursor
 type/behavior in R-Console? 
 She uses R 2.8.1, Windows 2000, and screenreader Virgo 4.6
 from Baum Retec,
 if that is any help.
 
 Your assistance with this problem and any other tips for
 teaching R to blind
 users will be much appreciated, 
 Rainer Scheuchenpflug
 
 Dr. Rainer Scheuchenpflug
 Lehrstuhl f?r Psychologie III
 R?ntgenring 11
 97070 W?rzburg
 Tel:   0931-31-82185
 Fax:   0931-31-82616
 Mail:  scheuchenpf...@psychologie.uni-wuerzburg.de
 Web:      http://www.izvw.de





__
R-help@r-project.org mailing list
https://stat.ethz.ch/mailman/listinfo/r-help
PLEASE do read the posting guide http://www.R-project.org/posting-guide.html
and provide commented, minimal, self-contained, reproducible code.


Re: [R] dot plot with error bands

2010-05-10 Thread Alexey Bessudnov
Many thanks for this suggestion. Indeed, labels turned out to be a 
factor, and after reordering the levels I got the plot I wanted.


Alexey


David Winsemius wrote:


On May 10, 2010, at 10:00 AM, Alexey Bessudnov wrote:


Dear all,

I'm trying to create a dot plot with error bands with

Dotplot(labels ~ Cbind(estimate, lower, upper), data=For.plot)

where estimate, lower and upper are numerical vectors, and labels is 
a character vector that contains labels.


The problem is that labels are automatically sorted alphabetically, 
and I want them to be sorted by estimate (as in my data frame). This 
should be straightforward, but unfortunately being new to R I can't 
figure out how to do this. I'll appreciate your guidance.


Have you tried making labels (an unfortunate choice for a variable 
name, BTW) a factor variable with levels in the order of your desire?


(Also being new to R, you may not recognize the difference between a 
factor variable and a character vector, so producing a more explicit 
description of the dataframe For.plot with the str function ought to 
be your next contribution to this thread if the above solution is not 
effective.




__
R-help@r-project.org mailing list
https://stat.ethz.ch/mailman/listinfo/r-help
PLEASE do read the posting guide http://www.R-project.org/posting-guide.html
and provide commented, minimal, self-contained, reproducible code.


[R] Polylogarithm

2010-05-10 Thread Jue (Andy) Wang
I am writing to ask if R has a build- in function to calculate this
polylogarithm Li_n(z) function , also known as the Jonquière's function
defined as

Li_n(z)=sum_(k=1)^infty(z^k)/(k^n)


Thanks

Andy

__
R-help@r-project.org mailing list
https://stat.ethz.ch/mailman/listinfo/r-help
PLEASE do read the posting guide http://www.R-project.org/posting-guide.html
and provide commented, minimal, self-contained, reproducible code.


[R] Dataframe horizontal scrolling

2010-05-10 Thread Michael H

R experts,
 
I am working with large multivariable data frames ( 50 variables) 
and I would like to scroll horizontally across my output
to view each data frame rather than having to scroll down vertically-
wrapped data frames.I have been using R Commander as a programming 
interface. If I assign a long character string to a vector I can scroll 
across its output easily but not a dataframe of equivalent width. I 
just want my data frame variables to fully output horizontally rather 
than partially with vertical wrapping, if that is possible. Any help 
would be appreciated.
 
Thank you,
 
Mike  
_
Hotmail has tools for the New Busy. Search, chat and e-mail from your inbox.

N:WL:en-US:WM_HMP:042010_1
__
R-help@r-project.org mailing list
https://stat.ethz.ch/mailman/listinfo/r-help
PLEASE do read the posting guide http://www.R-project.org/posting-guide.html
and provide commented, minimal, self-contained, reproducible code.


Re: [R] Supercripting text

2010-05-10 Thread Shang Gao
I tried writing that function before, but the expression() command is not 
preserved when we incorporate it into a function(). The first example you gave 
me, if run in R, produced a title that reads title instead of stuff. It 
seems that in this case the expression() can't read the character string in the 
argument specified in the function() command.

-Original Message-
From: Kevin Coombes [mailto:kevin.r.coom...@gmail.com] 
Sent: Monday, May 10, 2010 12:08 PM
To: Shang Gao
Cc: r-help@r-project.org
Subject: Re: [R] Supercripting text

You probably want to write your own titling function to add 
superscripts.  The following code will serve as a reasonable starting point:

# make the title expression
ourtitle - function(title, footnotes=NULL) {
  if (length(footnotes)  0) {
fn - paste(footnotes, collapse=' ')
title - eval(substitute(expression(bold(paste(title, ^fn))), 
list(fn=fn)))
  }
  title
}
# example with footnotes
tt - ourtitle(stuff, 1:3)
plot(1, 1)
title(tt)
# example without footnotes
plot(1, 1)
title(ourtitle(stuff))

Note that the only way I could make this work was to combine the 
footnote list before calling the eval-substitute-expression code; I 
cannot get it to work by applying things ot a list.

Kevin

Shang Gao wrote:
 Dear R users,

 I recently developed a plotting function in R and introduced it to my 
 coworkers. The function is designed to make plotting easier and more 
 efficient, which will in turn be more cost-effective for the company. The 
 reviews for the function have been positive thus far, except for one issue -- 
 addition of superscripts to the title. We need superscipts in the titles 
 sometimes to highlight footnotes which appear at the bottom of the plots.

 The syntax for supersciprts, however, is rather cumbersome, especially in 
 titles since it needs to be bolded. So far the only way of superscripting is 
 to use the expression() function. But to go about formatting the text such 
 that it appears bolded as a title in my plots, I would have to type in the 
 command
   
 text(expression(bold(paste(text for title,^1))))
 

 In some cases, the plot would require 3 footnotes to be shown, and the code 
 would be
   
 text(expression(bold(paste(text for title,^1,  ^2,  ^3))).)
 

 Most of my coworkers are still in the process of picking up R, some have 
 never used R before. The above commands may be a little too much for them to 
 handle.

 Is there an easier way of superscripting texts in R? It would be great if any 
 of you know of alternative ways other than using the expression() function.

 I greatly appreciate your help.

 Thank you.

 Sincerely,
 Shang


   [[alternative HTML version deleted]]

 __
 R-help@r-project.org mailing list
 https://stat.ethz.ch/mailman/listinfo/r-help
 PLEASE do read the posting guide http://www.R-project.org/posting-guide.html
 and provide commented, minimal, self-contained, reproducible code.
   

__
R-help@r-project.org mailing list
https://stat.ethz.ch/mailman/listinfo/r-help
PLEASE do read the posting guide http://www.R-project.org/posting-guide.html
and provide commented, minimal, self-contained, reproducible code.


[R] step function

2010-05-10 Thread Kim Jung Hwa
Hi All

I need some help with plotting a step function, currently I'm using

sfun - stepfun(c(1, 2, 5,10, 20), c(0, 11, 22, 33, 44, 0), f=0)
plot(sfun, pch=NA, main=, xlim=c(1,20))

which I working fine, but my data is in the following format:

Min Max Value
1 2 11
2 5 22
510 33
10 20 44

1. To save time, is there a way to directly plot the above data as it is
(without any reformatting).
2. Also, I need with starting the x-axis value from 1, xlim is not
completely helping me with this.

Any help would be highly appreciated. Thank you,
Kim

[[alternative HTML version deleted]]

__
R-help@r-project.org mailing list
https://stat.ethz.ch/mailman/listinfo/r-help
PLEASE do read the posting guide http://www.R-project.org/posting-guide.html
and provide commented, minimal, self-contained, reproducible code.


Re: [R] Dataframe horizontal scrolling

2010-05-10 Thread Ista Zahn
Hi Mike,
You can set options(width = x), where x equals the number of columns.

-Ista

On Monday 10 May 2010 16:06:54 Michael H wrote:
 R experts,
 
 I am working with large multivariable data frames ( 50 variables)
 and I would like to scroll horizontally across my output
 to view each data frame rather than having to scroll down vertically-
 wrapped data frames.I have been using R Commander as a programming
 interface. If I assign a long character string to a vector I can scroll
 across its output easily but not a dataframe of equivalent width. I
 just want my data frame variables to fully output horizontally rather
 than partially with vertical wrapping, if that is possible. Any help
 would be appreciated.
 
 Thank you,
 
 Mike
 _
 Hotmail has tools for the New Busy. Search, chat and e-mail from your
 inbox.
 
 N:WL:en-US:WM_HMP:042010_1
 __
 R-help@r-project.org mailing list
 https://stat.ethz.ch/mailman/listinfo/r-help
 PLEASE do read the posting guide
 http://www.R-project.org/posting-guide.html and provide commented,
 minimal, self-contained, reproducible code.

__
R-help@r-project.org mailing list
https://stat.ethz.ch/mailman/listinfo/r-help
PLEASE do read the posting guide http://www.R-project.org/posting-guide.html
and provide commented, minimal, self-contained, reproducible code.


[R] Corrupt R installation?

2010-05-10 Thread Ralf B
I installed the lattice package, and got an error that R was not able
to remove the previous version of lattice. Now my installation seems
to be currupt, even affecting other packages. I am getting this error
when loading TTR:

 library(TTR)
Loading required package: xts
Loading required package: zoo
Error in loadNamespace(i, c(lib.loc, .libPaths())) :
  there is no package called 'lattice'
In addition: Warning messages:
1: package 'TTR' was built under R version 2.10.1
2: package 'xts' was built under R version 2.10.1
3: package 'zoo' was built under R version 2.10.1
Error: package 'zoo' could not be loaded

My question now is, is there a way to manually remove lattice (or
whats left from it) ? Or do I have to go through the process of
completely re-installing? What do you guys do to prevent such a
situation - is there an easy way to secure a R installation?

Ralf

__
R-help@r-project.org mailing list
https://stat.ethz.ch/mailman/listinfo/r-help
PLEASE do read the posting guide http://www.R-project.org/posting-guide.html
and provide commented, minimal, self-contained, reproducible code.


[R] R algorithm/package for creating spatial autocorrelation of uniformly distributed landscape values

2010-05-10 Thread Laura S
Dear all:

I would like to create a landscape of environmental values that follow a
uniform frequency distribution and also have spatial autocorrelation in the
landscape. I was wondering if there is an algorithm and/or package out there
that creates autocorrelation of values that are distributed according to a
non-normal frequency distribution.

Any suggestions are greatly appreciated.

Thank you,
Laura

-- 
 Genius is the summed production of the many with the names of the few
attached for easy recall, unfairly so to other scientists

- E. O. Wilson (The Diversity of Life)

[[alternative HTML version deleted]]

__
R-help@r-project.org mailing list
https://stat.ethz.ch/mailman/listinfo/r-help
PLEASE do read the posting guide http://www.R-project.org/posting-guide.html
and provide commented, minimal, self-contained, reproducible code.


Re: [R] Dataframe horizontal scrolling

2010-05-10 Thread Erik Iverson
Perhaps even ?View would be useful here.  I've never used R Commander, 
so don't know it would be useful in that environment.


Michael H wrote:

R experts,
 
I am working with large multivariable data frames ( 50 variables) 
and I would like to scroll horizontally across my output

to view each data frame rather than having to scroll down vertically-
wrapped data frames.I have been using R Commander as a programming 
interface. If I assign a long character string to a vector I can scroll 
across its output easily but not a dataframe of equivalent width. I 
just want my data frame variables to fully output horizontally rather 
than partially with vertical wrapping, if that is possible. Any help 
would be appreciated.
 
Thank you,
 
Mike 		 	   		  
_

Hotmail has tools for the New Busy. Search, chat and e-mail from your inbox.

N:WL:en-US:WM_HMP:042010_1
__
R-help@r-project.org mailing list
https://stat.ethz.ch/mailman/listinfo/r-help
PLEASE do read the posting guide http://www.R-project.org/posting-guide.html
and provide commented, minimal, self-contained, reproducible code.


__
R-help@r-project.org mailing list
https://stat.ethz.ch/mailman/listinfo/r-help
PLEASE do read the posting guide http://www.R-project.org/posting-guide.html
and provide commented, minimal, self-contained, reproducible code.


[R] cumsum output

2010-05-10 Thread Felipe Carrillo
Hi: Thanks to Dennis and Fernando for your help reordering the levels.
Now I have a different issue:
I am trying to get the cumulative weekly values using cumsum and it
appears to output the wrong values. Here's my dataset: 

winter -  structure(list(week = c(26L, 27L, 28L, 29L, 30L, 31L, 32L, 33L, 
34L, 35L, 36L, 37L, 38L, 39L, 40L, 41L, 42L, 43L, 44L, 45L, 46L, 
47L, 48L, 49L, 50L, 51L, 52L, 1L, 2L, 3L, 4L, 5L, 6L, 7L, 8L, 
9L, 10L, 11L, 12L, 13L, 14L, 15L, 16L, 17L, 18L, 19L, 20L, 21L, 
22L, 23L, 24L, 25L), BY2009 = c(0L, 95L, 436L, 809L, 3668L, 9437L, 
42700L, 141135L, 486474L, 1109095L, 1459053L, 1990285L, 2643088L, 
2988446L, 3131437L, 3280320L, 4237821L, 4270316L, 4285178L, 4292388L, 
4302153L, 4308999L, 4314066L, 4320267L, 4323312L, 4429542L, 4430725L, 
4430920L, 4437775L, 4448036L, 4452215L, 4452865L, 4453105L, 4453270L, 
4454459L, 4455322L, 4455884L, 4457234L, 4457422L, 4457901L, 4458671L, 
4458671L, 4459229L, 4459543L, 4459690L, NA, NA, NA, NA, NA, NA, 
NA), BY2008 = c(0L, 0L, 77L, 201L, 360L, 2736L, 12216L, 29530L, 
129104L, 452783L, 650994L, 744624L, 873807L, 985627L, 1056887L, 
1092128L, 1106148L, 1126926L, 1148620L, 1163636L, 1177036L, 1186223L, 
1189830L, 1192634L, 1195051L, 1200342L, 1216558L, 1217456L, 1218014L, 
1218723L, 1219695L, 1219756L, 1220128L, 1223214L, 1233322L, 1237617L, 
1238499L, 1241092L, 1241128L, 1241361L, 1241604L, 1241604L, 1241674L, 
1241946L, 1242254L, 1242388L, 1242428L, 1242428L, 1242428L, 1242428L, 
1242428L, 1242428L), BY2007 = c(0L, 0L, 0L, 10775L, 14941L, 19899L, 
36120L, 65521L, 100472L, 133660L, 279704L, 384711L, 570008L, 
729690L, 937227L, 1077921L, 1206196L, 1250470L, 1277549L, 1296477L, 
1306914L, 1336898L, 1355293L, 1381139L, 1385712L, 1417707L, 1421386L, 
1422429L, 1432065L, 1433589L, 1434416L, 1441425L, 1441658L, 1442091L, 
1442091L, 1443194L, 1443451L, 1443579L, 1443645L, 1443715L, 1444250L, 
1444679L, 1444776L, 1444776L, 1444776L, 1444776L, 1444776L, 1444776L, 
1444776L, 1444776L, 1444776L, 1444776L), BY2006 = c(0L, 707L, 
3097L, 11957L, 36387L, 77272L, 150064L, 355585L, 700078L, 1363051L, 
1889460L, 2521413L, 3371904L, 4214582L, 4660569L, 5218721L, 5550753L, 
5725079L, 5805680L, 5854376L, 5952947L, 6056510L, 6205979L, 6284060L, 
6466538L, 6468696L, 6485262L, 6489289L, 6491944L, 6493056L, 6493623L, 
6496218L, 6501194L, 6507530L, 6507824L, 6509582L, 6509873L, 6510076L, 
6510526L, 6511624L, 6512412L, 6512607L, 6513139L, 6513139L, 6513139L, 
6513139L, 6513139L, 6513139L, 6513139L, 6513306L, 6513306L, 6513306L
), BY2005 = c(0L, 0L, 868L, 2912L, 6976L, 13025L, 22424L, 35728L, 
80900L, 323055L, 799919L, 1512453L, 2570862L, 4685880L, 6196222L, 
7334435L, 7667627L, 7826447L, 7920826L, 8269708L, 8308998L, 8338699L, 
8385957L, 8455794L, 8463678L, 8513016L, 8535184L, 8554581L, 8573978L, 
8589962L, 8598650L, 8599850L, 8601473L, 8602764L, 8604120L, 8604827L, 
8605702L, 8606577L, 8606799L, 8607682L, 8607682L, 8607682L, 8607811L, 
8607811L, 8607811L, 8607811L, 8607811L, 8607811L, 8607811L, 8607811L, 
8607811L, 8607811L)), .Names = c(week, BY2009, BY2008, 
BY2007, BY2006, BY2005), class = data.frame, row.names = c(NA, 
-52L))
apply(winter[,2:6],2,cumsum)

 
Felipe D. Carrillo
Supervisory Fishery Biologist
Department of the Interior
US Fish  Wildlife Service
California, USA




__
R-help@r-project.org mailing list
https://stat.ethz.ch/mailman/listinfo/r-help
PLEASE do read the posting guide http://www.R-project.org/posting-guide.html
and provide commented, minimal, self-contained, reproducible code.


Re: [R] Corrupt R installation?

2010-05-10 Thread David Winsemius


On May 10, 2010, at 4:46 PM, Ralf B wrote:


I installed the lattice package, and got an error that R was not able
to remove the previous version of lattice. Now my installation seems
to be currupt, even affecting other packages. I am getting this error
when loading TTR:


library(TTR)

Loading required package: xts
Loading required package: zoo
Error in loadNamespace(i, c(lib.loc, .libPaths())) :
 there is no package called 'lattice'
In addition: Warning messages:
1: package 'TTR' was built under R version 2.10.1
2: package 'xts' was built under R version 2.10.1
3: package 'zoo' was built under R version 2.10.1
Error: package 'zoo' could not be loaded

My question now is, is there a way to manually remove lattice (or
whats left from it) ? Or do I have to go through the process of
completely re-installing? What do you guys do to prevent such a
situation - is there an easy way to secure a R installation?


You could try simply deleting the .Rdta file that is perhaps invisible  
to your file manager software. on whatever OS you are using.


--
David Winsemius, MD
West Hartford, CT

__
R-help@r-project.org mailing list
https://stat.ethz.ch/mailman/listinfo/r-help
PLEASE do read the posting guide http://www.R-project.org/posting-guide.html
and provide commented, minimal, self-contained, reproducible code.


Re: [R] cumsum output

2010-05-10 Thread jim holtman
It is working correctly,  You have NAs in your data, so you get NAs in the
cumsum.

On Mon, May 10, 2010 at 5:09 PM, Felipe Carrillo
mazatlanmex...@yahoo.comwrote:

 Hi: Thanks to Dennis and Fernando for your help reordering the levels.
 Now I have a different issue:
 I am trying to get the cumulative weekly values using cumsum and it
 appears to output the wrong values. Here's my dataset:

 winter -  structure(list(week = c(26L, 27L, 28L, 29L, 30L, 31L, 32L, 33L,
 34L, 35L, 36L, 37L, 38L, 39L, 40L, 41L, 42L, 43L, 44L, 45L, 46L,
 47L, 48L, 49L, 50L, 51L, 52L, 1L, 2L, 3L, 4L, 5L, 6L, 7L, 8L,
 9L, 10L, 11L, 12L, 13L, 14L, 15L, 16L, 17L, 18L, 19L, 20L, 21L,
 22L, 23L, 24L, 25L), BY2009 = c(0L, 95L, 436L, 809L, 3668L, 9437L,
 42700L, 141135L, 486474L, 1109095L, 1459053L, 1990285L, 2643088L,
 2988446L, 3131437L, 3280320L, 4237821L, 4270316L, 4285178L, 4292388L,
 4302153L, 4308999L, 4314066L, 4320267L, 4323312L, 4429542L, 4430725L,
 4430920L, 4437775L, 4448036L, 4452215L, 4452865L, 4453105L, 4453270L,
 4454459L, 4455322L, 4455884L, 4457234L, 4457422L, 4457901L, 4458671L,
 4458671L, 4459229L, 4459543L, 4459690L, NA, NA, NA, NA, NA, NA,
 NA), BY2008 = c(0L, 0L, 77L, 201L, 360L, 2736L, 12216L, 29530L,
 129104L, 452783L, 650994L, 744624L, 873807L, 985627L, 1056887L,
 1092128L, 1106148L, 1126926L, 1148620L, 1163636L, 1177036L, 1186223L,
 1189830L, 1192634L, 1195051L, 1200342L, 1216558L, 1217456L, 1218014L,
 1218723L, 1219695L, 1219756L, 1220128L, 1223214L, 1233322L, 1237617L,
 1238499L, 1241092L, 1241128L, 1241361L, 1241604L, 1241604L, 1241674L,
 1241946L, 1242254L, 1242388L, 1242428L, 1242428L, 1242428L, 1242428L,
 1242428L, 1242428L), BY2007 = c(0L, 0L, 0L, 10775L, 14941L, 19899L,
 36120L, 65521L, 100472L, 133660L, 279704L, 384711L, 570008L,
 729690L, 937227L, 1077921L, 1206196L, 1250470L, 1277549L, 1296477L,
 1306914L, 1336898L, 1355293L, 1381139L, 1385712L, 1417707L, 1421386L,
 1422429L, 1432065L, 1433589L, 1434416L, 1441425L, 1441658L, 1442091L,
 1442091L, 1443194L, 1443451L, 1443579L, 1443645L, 1443715L, 1444250L,
 1444679L, 1444776L, 1444776L, 1444776L, 1444776L, 1444776L, 1444776L,
 1444776L, 1444776L, 1444776L, 1444776L), BY2006 = c(0L, 707L,
 3097L, 11957L, 36387L, 77272L, 150064L, 355585L, 700078L, 1363051L,
 1889460L, 2521413L, 3371904L, 4214582L, 4660569L, 5218721L, 5550753L,
 5725079L, 5805680L, 5854376L, 5952947L, 6056510L, 6205979L, 6284060L,
 6466538L, 6468696L, 6485262L, 6489289L, 6491944L, 6493056L, 6493623L,
 6496218L, 6501194L, 6507530L, 6507824L, 6509582L, 6509873L, 6510076L,
 6510526L, 6511624L, 6512412L, 6512607L, 6513139L, 6513139L, 6513139L,
 6513139L, 6513139L, 6513139L, 6513139L, 6513306L, 6513306L, 6513306L
 ), BY2005 = c(0L, 0L, 868L, 2912L, 6976L, 13025L, 22424L, 35728L,
 80900L, 323055L, 799919L, 1512453L, 2570862L, 4685880L, 6196222L,
 7334435L, 7667627L, 7826447L, 7920826L, 8269708L, 8308998L, 8338699L,
 8385957L, 8455794L, 8463678L, 8513016L, 8535184L, 8554581L, 8573978L,
 8589962L, 8598650L, 8599850L, 8601473L, 8602764L, 8604120L, 8604827L,
 8605702L, 8606577L, 8606799L, 8607682L, 8607682L, 8607682L, 8607811L,
 8607811L, 8607811L, 8607811L, 8607811L, 8607811L, 8607811L, 8607811L,
 8607811L, 8607811L)), .Names = c(week, BY2009, BY2008,
 BY2007, BY2006, BY2005), class = data.frame, row.names = c(NA,
 -52L))
 apply(winter[,2:6],2,cumsum)


 Felipe D. Carrillo
 Supervisory Fishery Biologist
 Department of the Interior
 US Fish  Wildlife Service
 California, USA




 __
 R-help@r-project.org mailing list
 https://stat.ethz.ch/mailman/listinfo/r-help
 PLEASE do read the posting guide
 http://www.R-project.org/posting-guide.htmlhttp://www.r-project.org/posting-guide.html
 and provide commented, minimal, self-contained, reproducible code.




-- 
Jim Holtman
Cincinnati, OH
+1 513 646 9390

What is the problem that you are trying to solve?

[[alternative HTML version deleted]]

__
R-help@r-project.org mailing list
https://stat.ethz.ch/mailman/listinfo/r-help
PLEASE do read the posting guide http://www.R-project.org/posting-guide.html
and provide commented, minimal, self-contained, reproducible code.


Re: [R] cumsum output

2010-05-10 Thread Felipe Carrillo
I realized that after I hit the send button...I was looking at the wrong 
dataset,Brr
 
Felipe D. Carrillo
Supervisory Fishery Biologist
Department of the Interior
US Fish  Wildlife Service
California, USA



From: jim holtman jholt...@gmail.com
To: Felipe Carrillo mazatlanmex...@yahoo.com
Cc: r-h...@stat.math.ethz.ch
Sent: Mon, May 10, 2010 2:33:03 PM
Subject: Re: [R] cumsum output

It is working correctly,  You have NAs in your data, so you get NAs in the 
cumsum.


On Mon, May 10, 2010 at 5:09 PM, Felipe Carrillo mazatlanmex...@yahoo.com 
wrote:

Hi: Thanks to Dennis and Fernando for your help reordering the levels.
Now I have a different issue:
I am trying to get the cumulative weekly values using cumsum and it
appears to output the wrong values. Here's my dataset:

winter -  structure(list(week = c(26L, 27L, 28L, 29L, 30L, 31L, 32L, 33L,
34L, 35L, 36L, 37L, 38L, 39L, 40L, 41L, 42L, 43L, 44L, 45L, 46L,
47L, 48L, 49L, 50L, 51L, 52L, 1L, 2L, 3L, 4L, 5L, 6L, 7L, 8L,
9L, 10L, 11L, 12L, 13L, 14L, 15L, 16L, 17L, 18L, 19L, 20L, 21L,
22L, 23L, 24L, 25L), BY2009 = c(0L, 95L, 436L, 809L, 3668L, 9437L,
42700L, 141135L, 486474L, 1109095L, 1459053L, 1990285L, 2643088L,
2988446L, 3131437L, 3280320L, 4237821L, 4270316L, 4285178L, 4292388L,
4302153L, 4308999L, 4314066L, 4320267L, 4323312L, 4429542L, 4430725L,
4430920L, 4437775L, 4448036L, 4452215L, 4452865L, 4453105L, 4453270L,
4454459L, 4455322L, 4455884L, 4457234L, 4457422L, 4457901L, 4458671L,
4458671L, 4459229L, 4459543L, 4459690L, NA, NA, NA, NA, NA, NA,
NA), BY2008 = c(0L, 0L, 77L, 201L, 360L, 2736L, 12216L, 29530L,
129104L, 452783L, 650994L, 744624L, 873807L, 985627L, 1056887L,
1092128L, 1106148L, 1126926L, 1148620L, 1163636L, 1177036L, 1186223L,
1189830L, 1192634L, 1195051L, 1200342L, 1216558L, 1217456L, 1218014L,
1218723L, 1219695L, 1219756L, 1220128L, 1223214L, 1233322L, 1237617L,
1238499L, 1241092L, 1241128L, 1241361L, 1241604L, 1241604L, 1241674L,
1241946L, 1242254L, 1242388L, 1242428L, 1242428L, 1242428L, 1242428L,
1242428L, 1242428L), BY2007 = c(0L, 0L, 0L, 10775L, 14941L, 19899L,
36120L, 65521L, 100472L, 133660L, 279704L, 384711L, 570008L,
729690L, 937227L, 1077921L, 1206196L, 1250470L, 1277549L, 1296477L,
1306914L, 1336898L, 1355293L, 1381139L, 1385712L, 1417707L, 1421386L,
1422429L, 1432065L, 1433589L, 1434416L, 1441425L, 1441658L, 1442091L,
1442091L, 1443194L, 1443451L, 1443579L, 1443645L, 1443715L, 1444250L,
1444679L, 1444776L, 1444776L, 1444776L, 1444776L, 1444776L, 1444776L,
1444776L, 1444776L, 1444776L, 1444776L), BY2006 = c(0L, 707L,
3097L, 11957L, 36387L, 77272L, 150064L, 355585L, 700078L, 1363051L,
1889460L, 2521413L, 3371904L, 4214582L, 4660569L, 5218721L, 5550753L,
5725079L, 5805680L, 5854376L, 5952947L, 6056510L, 6205979L, 6284060L,
6466538L, 6468696L, 6485262L, 6489289L, 6491944L, 6493056L, 6493623L,
6496218L, 6501194L, 6507530L, 6507824L, 6509582L, 6509873L, 6510076L,
6510526L, 6511624L, 6512412L, 6512607L, 6513139L, 6513139L, 6513139L,
6513139L, 6513139L, 6513139L, 6513139L, 6513306L, 6513306L, 6513306L
), BY2005 = c(0L, 0L, 868L, 2912L, 6976L, 13025L, 22424L, 35728L,
80900L, 323055L, 799919L, 1512453L, 2570862L, 4685880L, 6196222L,
7334435L, 7667627L, 7826447L, 7920826L, 8269708L, 8308998L, 8338699L,
8385957L, 8455794L, 8463678L, 8513016L, 8535184L, 8554581L, 8573978L,
8589962L, 8598650L, 8599850L, 8601473L, 8602764L, 8604120L, 8604827L,
8605702L, 8606577L, 8606799L, 8607682L, 8607682L, 8607682L, 8607811L,
8607811L, 8607811L, 8607811L, 8607811L, 8607811L, 8607811L, 8607811L,
8607811L, 8607811L)), .Names = c(week, BY2009, BY2008,
BY2007, BY2006, BY2005), class = data.frame, row.names = c(NA,
-52L))
apply(winter[,2:6],2,cumsum)

 
Felipe D. Carrillo
Supervisory Fishery Biologist
Department of the Interior
US Fish  Wildlife Service
California, USA




__
R-help@r-project.org mailing list
https://stat.ethz.ch/mailman/listinfo/r-help
PLEASE do read the posting guide http://www.r-project.org/posting-guide.html
and provide commented, minimal, self-contained, reproducible code.



-- 
Jim Holtman
Cincinnati, OH
+1 513 646 9390

What is the problem that you are trying to solve?



  
[[alternative HTML version deleted]]

__
R-help@r-project.org mailing list
https://stat.ethz.ch/mailman/listinfo/r-help
PLEASE do read the posting guide http://www.R-project.org/posting-guide.html
and provide commented, minimal, self-contained, reproducible code.


Re: [R] [Fwd: Re: Plotting log-axis with the exponential base to

2010-05-10 Thread Ted Harding
Elisabeth, question to you: How is it that you recognise that it is
a logaritmic axis with the base of 10, as opposed to any other base?
Ted.

On 10-May-10 17:15:04, Elisabeth Bjerke Rastad wrote:
 Hello!
 Thank you for answering!
 What I am trying to do is to plot my raw values (biomass of different
 species) on a logaritmic y-axis with the base of e. When I type
 log=y,
 the axis transforms into a logaritmic axis with the base of 10.
 
 Best regards,
 Elisabeth
 
 Dear Elisabeth,

 I'm not sure if I have understood your question -- are you trying
 to use a different logarithmic base?

  Kind regards,

  Xianwen

 On Sun, May 9, 2010 at 8:05 PM, Elisabeth Bjerke Rastad
 ebr...@post.uit.no wrote:
 Hello!

 I have a problem which I have tried to solve for several days now..

 I have plottet a lineplot.CI in the library sciplot, and I am
 trying
 to
 plot it with a logaritmic y-axis (with exponential base).

 The problem is that; when I type log y, the axis transforms into
 the
 logaritmic of base 10.

 I wonder if someeone could tell me how to specify that I would like
 to
 use
 the exponential logaritmic y-axis. I have tried a lot (but obviously
 not
 all, I guess this problem is possible to solve..)

 Hope you would like to help me! Thank you a lot in advance!!

 Greetings,

 Elisabeth B. RÃ¥stad
 (Master's student, Norway)

 __
 R-help@r-project.org mailing list
 https://stat.ethz.ch/mailman/listinfo/r-help
 PLEASE do read the posting guide
 http://www.R-project.org/posting-guide.html
 and provide commented, minimal, self-contained, reproducible code.




 --
 Xianwen Chen, M.Sc.
 Scientific assistant, BFE, UiT (www.uit.no)
 Tel.: +47 776 46 112 | Fax: +47 776 46 020


 
 __
 R-help@r-project.org mailing list
 https://stat.ethz.ch/mailman/listinfo/r-help
 PLEASE do read the posting guide
 http://www.R-project.org/posting-guide.html
 and provide commented, minimal, self-contained, reproducible code.


E-Mail: (Ted Harding) ted.hard...@manchester.ac.uk
Fax-to-email: +44 (0)870 094 0861
Date: 10-May-10   Time: 22:48:35
-- XFMail --

__
R-help@r-project.org mailing list
https://stat.ethz.ch/mailman/listinfo/r-help
PLEASE do read the posting guide http://www.R-project.org/posting-guide.html
and provide commented, minimal, self-contained, reproducible code.


[R] Robust SE Heteroskedasticity-consistent estimation

2010-05-10 Thread RATIARISON Eric
Hi,

I'm using maxlik with functions specified (L, his gradient  hessian).

Now I would like determine some robust standard errors of my estimators.

So I 'm try to use vcovHC, or hccm or robcov for example

 but in use one of them with my result of maxlik, I've a the following
error message :

 

Erreur dans terms.default(object) : no terms component

 

Is there some attributes to give to maxlik objet for fitting the call
of vcovHC?

 

Thank you

 


[[alternative HTML version deleted]]

__
R-help@r-project.org mailing list
https://stat.ethz.ch/mailman/listinfo/r-help
PLEASE do read the posting guide http://www.R-project.org/posting-guide.html
and provide commented, minimal, self-contained, reproducible code.


Re: [R] Dataframe horizontal scrolling

2010-05-10 Thread Emmanuel Charpentier
under Unix/X11 (and IIRC under Windows), edit(some.data.frame) will
invoke a smallish spreadsheet-like data editor, which will allow you to
move around your data with no fuss. I probably wouldn't use it for any
serious data entry, but found damn useful in a lot of data-debugging
situations (you know, the sort of ... thing ... that hit the fan when
you've been coaxed to accept Excel spreatsheet as dats sets).

HTH,

Emmanuel Charpentier

Le lundi 10 mai 2010 à 16:06 -0400, Michael H a écrit :
 R experts,
  
 I am working with large multivariable data frames ( 50 variables) 
 and I would like to scroll horizontally across my output
 to view each data frame rather than having to scroll down vertically-
 wrapped data frames.I have been using R Commander as a programming 
 interface. If I assign a long character string to a vector I can scroll 
 across its output easily but not a dataframe of equivalent width. I 
 just want my data frame variables to fully output horizontally rather 
 than partially with vertical wrapping, if that is possible. Any help 
 would be appreciated.
  
 Thank you,
  
 Mike
 _
 Hotmail has tools for the New Busy. Search, chat and e-mail from your inbox.
 
 N:WL:en-US:WM_HMP:042010_1

__
R-help@r-project.org mailing list
https://stat.ethz.ch/mailman/listinfo/r-help
PLEASE do read the posting guide http://www.R-project.org/posting-guide.html
and provide commented, minimal, self-contained, reproducible code.


[R] cex.axis on levelplot

2010-05-10 Thread vjaneiro


I'm trying to use cex.axis or something like this to augment the size
labels' on levelplot axis, but it doesn't work.

Could someone help me? 
-- 
View this message in context: 
http://r.789695.n4.nabble.com/cex-axis-on-levelplot-tp2172879p2172879.html
Sent from the R help mailing list archive at Nabble.com.

[[alternative HTML version deleted]]

__
R-help@r-project.org mailing list
https://stat.ethz.ch/mailman/listinfo/r-help
PLEASE do read the posting guide http://www.R-project.org/posting-guide.html
and provide commented, minimal, self-contained, reproducible code.


[R] predict() without generating the model within R

2010-05-10 Thread Chaudhari, Bimal
Is there a predict method/syntax which I could use to generate
predictions (and other output from predict() methods) if I have the
model parameter estimates from a training dataset but not the data used
to generate the original model (the models were generated by a
collaborator using STATA and for IRB reasons I am not allowed
independent access to the original data)?  I have the new/testing data
upon which I wish to base my predictions.

Thanks,
Bimal

PS-yes, I know that if my collaborator just used R, he could send me
just the model as an Rdata object.  That is not going to happen in this
instance.


Bimal P Chaudhari, MPH
Boston University
MD Candidate, 2011
Washington University in St. Louis
MS Candidate, 2010
Doris Duke Clinical Research Fellow
314-286-2864

__
R-help@r-project.org mailing list
https://stat.ethz.ch/mailman/listinfo/r-help
PLEASE do read the posting guide http://www.R-project.org/posting-guide.html
and provide commented, minimal, self-contained, reproducible code.


[R] plotting data when all you have is the summary data

2010-05-10 Thread Chaudhari, Bimal
Are there functions/packages which support plots (bar and/or line) where
I provide the point estimate and some error measure rather than the raw
data?

I often have to summarize/present data from multiple sources where the
original data is unavailable and do not yet have a good solution for
this.

Thanks,
bimal

Bimal P Chaudhari, MPH
Boston University
MD Candidate, 2011
Washington University in St. Louis
MS Candidate, 2010
Doris Duke Clinical Research Fellow
314-286-2864

__
R-help@r-project.org mailing list
https://stat.ethz.ch/mailman/listinfo/r-help
PLEASE do read the posting guide http://www.R-project.org/posting-guide.html
and provide commented, minimal, self-contained, reproducible code.


Re: [R] predict() without generating the model within R

2010-05-10 Thread Duncan Murdoch

On 10/05/2010 6:13 PM, Chaudhari, Bimal wrote:

Is there a predict method/syntax which I could use to generate
predictions (and other output from predict() methods) if I have the
model parameter estimates from a training dataset but not the data used
to generate the original model (the models were generated by a
collaborator using STATA and for IRB reasons I am not allowed
independent access to the original data)?  I have the new/testing data
upon which I wish to base my predictions.
  


That depends on what model is being fit, but in general I'd say it would 
be impractical to put together enough information for predict() to work 
properly.  However, you can do predictions for many models just from 
their definition, and you may have enough information to calculate 
errors as well.  But you'll need to program the formulas yourself.


Duncan Murdoch

Thanks,
Bimal

PS-yes, I know that if my collaborator just used R, he could send me
just the model as an Rdata object.  That is not going to happen in this
instance.


Bimal P Chaudhari, MPH
Boston University
MD Candidate, 2011
Washington University in St. Louis
MS Candidate, 2010
Doris Duke Clinical Research Fellow
314-286-2864

__
R-help@r-project.org mailing list
https://stat.ethz.ch/mailman/listinfo/r-help
PLEASE do read the posting guide http://www.R-project.org/posting-guide.html
and provide commented, minimal, self-contained, reproducible code.



__
R-help@r-project.org mailing list
https://stat.ethz.ch/mailman/listinfo/r-help
PLEASE do read the posting guide http://www.R-project.org/posting-guide.html
and provide commented, minimal, self-contained, reproducible code.


Re: [R] cex.axis on levelplot

2010-05-10 Thread David Winsemius


On May 10, 2010, at 4:29 PM, vjaneiro wrote:




I'm trying to use cex.axis or something like this to augment the size
labels' on levelplot axis, but it doesn't work.

Could someone help me?


Look at the thread Re: [R] Increasing the font size on axes in  
trellis from two days ago.



--



David Winsemius, MD
West Hartford, CT

__
R-help@r-project.org mailing list
https://stat.ethz.ch/mailman/listinfo/r-help
PLEASE do read the posting guide http://www.R-project.org/posting-guide.html
and provide commented, minimal, self-contained, reproducible code.


Re: [R] plotting data when all you have is the summary data

2010-05-10 Thread David Winsemius


On May 10, 2010, at 6:17 PM, Chaudhari, Bimal wrote:

Are there functions/packages which support plots (bar and/or line)  
where
I provide the point estimate and some error measure rather than the  
raw

data?

I often have to summarize/present data from multiple sources where the
original data is unavailable and do not yet have a good solution for
this.


There are R packages that support meta analysis. Learn to search,  
grasshopper.




Thanks,
bimal

Bimal P Chaudhari, MPH
Boston University
MD Candidate, 2011
Washington University in St. Louis
MS Candidate, 2010



David Winsemius, MD, MPH
West Hartford, CT

__
R-help@r-project.org mailing list
https://stat.ethz.ch/mailman/listinfo/r-help
PLEASE do read the posting guide http://www.R-project.org/posting-guide.html
and provide commented, minimal, self-contained, reproducible code.


Re: [R] Robust SE Heteroskedasticity-consistent estimation

2010-05-10 Thread Achim Zeileis

On Mon, 10 May 2010, RATIARISON Eric wrote:


Hi,

I'm using maxlik with functions specified (L, his gradient  hessian).

Now I would like determine some robust standard errors of my estimators.

So I 'm try to use vcovHC, or hccm or robcov for example

but in use one of them with my result of maxlik, I've a the following
error message :

Erreur dans terms.default(object) : no terms component

Is there some attributes to give to maxlik objet for fitting the call
of vcovHC?


This is discussed in
  vignette(sandwich-OOP, package = sandwich)
one of the vignettes accompanying the sandwich package that provides the 
vcovHC() function. At the very least, you need an estfun() method which 
extracts the gradient contributions per observation. Then you need a 
bread() function, typically based on the observed Hessian. Then you can 
compute the basic sandwich() estimators.


Best,
Z




Thank you




[[alternative HTML version deleted]]

__
R-help@r-project.org mailing list
https://stat.ethz.ch/mailman/listinfo/r-help
PLEASE do read the posting guide http://www.R-project.org/posting-guide.html
and provide commented, minimal, self-contained, reproducible code.



__
R-help@r-project.org mailing list
https://stat.ethz.ch/mailman/listinfo/r-help
PLEASE do read the posting guide http://www.R-project.org/posting-guide.html
and provide commented, minimal, self-contained, reproducible code.


Re: [R] System neutral Daylight Savings Time response?

2010-05-10 Thread Gabor Grothendieck
Perhaps you should be using the chron package.  It has no time zones
in the first place.

On Mon, May 10, 2010 at 2:26 PM, Garrett Grolemund g...@rice.edu wrote:
 I'm searching for an r command that will notify me if I create a time that
 does not exist due to Daylight Savings Time. For example, if I run the
 following command on a windows machine

 ISOdatetime(2010,03,14,2,10,0, tz = ) # My system time is set to the
 United States Central Time Zone
 [1] NA

 R returns NA, which is the behavior I want. However, if I run the same
 command on a mac, R returns a POSIXct object and I have to examine the
 object manually to notice that I had tried to create an impossible time.

 ISOdatetime(2010,03,14,2,10,0, tz = )
 [1] 2010-03-14 01:10:00 CST

 Is there a method of creating time objects in R that will always return NA
 for non-existant times, no matter the operating system?

 Thank you sincerely,
 Garrett

        [[alternative HTML version deleted]]

 __
 R-help@r-project.org mailing list
 https://stat.ethz.ch/mailman/listinfo/r-help
 PLEASE do read the posting guide http://www.R-project.org/posting-guide.html
 and provide commented, minimal, self-contained, reproducible code.


__
R-help@r-project.org mailing list
https://stat.ethz.ch/mailman/listinfo/r-help
PLEASE do read the posting guide http://www.R-project.org/posting-guide.html
and provide commented, minimal, self-contained, reproducible code.


Re: [R] R algorithm/package for creating spatial autocorrelation of uniformly distributed landscape values

2010-05-10 Thread David Winsemius


On May 10, 2010, at 4:54 PM, Laura S wrote:


Dear all:

I would like to create a landscape of environmental values that  
follow a
uniform frequency distribution and also have spatial autocorrelation  
in the

landscape.


Perhaps:
X - distribution on one dimension
Y - distribution on another dimension
cY - rho*X +(1-rho)*Y



I was wondering if there is an algorithm and/or package out there
that creates autocorrelation of values that are distributed  
according to a

non-normal frequency distribution.


There are several packages with the letters distr as a substring.  
And there is also at least one package that will assist in creating  
copulas.




Any suggestions are greatly appreciated.

Thank you,
Laura

--
 Genius is the summed production of the many with the names of the  
few

attached for easy recall, unfairly so to other scientists

- E. O. Wilson (The Diversity of Life)

[[alternative HTML version deleted]]

__
R-help@r-project.org mailing list
https://stat.ethz.ch/mailman/listinfo/r-help
PLEASE do read the posting guide http://www.R-project.org/posting-guide.html
and provide commented, minimal, self-contained, reproducible code.


David Winsemius, MD
West Hartford, CT

__
R-help@r-project.org mailing list
https://stat.ethz.ch/mailman/listinfo/r-help
PLEASE do read the posting guide http://www.R-project.org/posting-guide.html
and provide commented, minimal, self-contained, reproducible code.


[R] ggplot: Trouble with xlim() and discrete scales

2010-05-10 Thread John Rauser
I'm learning ggplot and am a little confused. Sometimes discrete scales work
like I'd expect, and sometimes they don't.  For example...

This works exactly like one would expect:

df-data.frame(names=c(Bob,Mary,Joe,Bob,Bob))
ggplot(df,aes(names))+geom_histogram()

But this yields an error:

ggplot(df,aes(names))+geom_histogram()+xlim(Bob)
Error in data.frame(count = as.numeric(tapply(weight, bins, sum, na.rm =
TRUE))$
  arguments imply differing number of rows: 0, 1

...as does this:

ggplot(df,aes(names))+geom_histogram()+xlim(Bob,Mary)

... but this works fine:

ggplot(df,aes(names))+geom_histogram()+xlim(Bob,Mary,Joe)

... and curiously, so does this:

ggplot(df,aes(names))+geom_histogram()+xlim(Bob,Mary,Joe,Frank)

... and even more confusingly, this works perfectly:

ggplot(df,aes(names,..density..,group=1))+geom_histogram()+xlim(Bob,Mary)


This feels like a bug, but perhaps I'm doing something dumb.  Can anyone
clarify?


And while I have your attention: as a ggplot novice, I often find myself
getting cryptic error messages like the one above.  Nearly always this is
because I'm asking it to do something unreasonable, but it often takes me
quite a long time to figure out my error.  Does anyone have general tips for
debugging ggplot commands?  Anything better than summary(p)?


Thanks very much for your help,

-J

[[alternative HTML version deleted]]

__
R-help@r-project.org mailing list
https://stat.ethz.ch/mailman/listinfo/r-help
PLEASE do read the posting guide http://www.R-project.org/posting-guide.html
and provide commented, minimal, self-contained, reproducible code.


Re: [R] ggplot: Trouble with xlim() and discrete scales

2010-05-10 Thread David Winsemius


On May 10, 2010, at 7:36 PM, John Rauser wrote:

I'm learning ggplot and am a little confused. Sometimes discrete  
scales work

like I'd expect, and sometimes they don't.  For example...

This works exactly like one would expect:

df-data.frame(names=c(Bob,Mary,Joe,Bob,Bob))
ggplot(df,aes(names))+geom_histogram()

But this yields an error:

ggplot(df,aes(names))+geom_histogram()+xlim(Bob)
Error in data.frame(count = as.numeric(tapply(weight, bins, sum,  
na.rm =

TRUE))$
 arguments imply differing number of rows: 0, 1


Well, a range with only one value seems a bit  ... degenerate?


...as does this:

ggplot(df,aes(names))+geom_histogram()+xlim(Bob,Mary)



 str(df)
'data.frame':   5 obs. of  1 variable:
 $ names: Factor w/ 3 levels Bob,Joe,Mary: 1 3 2 1 1


... but this works fine:

ggplot(df,aes(names))+geom_histogram()+xlim(Bob,Mary,Joe)


So now that range covers the factor's possible values, and it  
appears Hadley's coding is willing to a accept a degree of abstraction.




... and curiously, so does this:

ggplot(df,aes(names))+geom_histogram() 
+xlim(Bob,Mary,Joe,Frank)


... and even more confusingly, this works perfectly:

ggplot(df,aes(names,..density..,group=1))+geom_histogram() 
+xlim(Bob,Mary)



This feels like a bug, but perhaps I'm doing something dumb.


Looks dumb to me.


Can anyone
clarify?


And while I have your attention: as a ggplot novice, I often find  
myself
getting cryptic error messages like the one above.  Nearly always  
this is
because I'm asking it to do something unreasonable, but it often  
takes me
quite a long time to figure out my error.  Does anyone have general  
tips for

debugging ggplot commands?  Anything better than summary(p)?


Use str to look at your data. gives you further information about  
classes.



David Winsemius, MD
West Hartford, CT

__
R-help@r-project.org mailing list
https://stat.ethz.ch/mailman/listinfo/r-help
PLEASE do read the posting guide http://www.R-project.org/posting-guide.html
and provide commented, minimal, self-contained, reproducible code.


Re: [R] ggplot: Trouble with xlim() and discrete scales

2010-05-10 Thread John Rauser
May I ask for further illumination of my dumbness?

I had been merrily plotting ..density.. and using xlim() to constrain the
x-axis like this and everything was working fine:

df-data.frame(names=c(Bob,Mary,Joe,Bob,Bob))
ggplot(df,aes(names,..density..,group=1))+geom_histogram()+xlim(Bob,Mary)

So I was a little surprised when this didn't work the same way:

ggplot(df,aes(names))+geom_histogram()+xlim(Bob,Mary)

Actually, looking at the above sparks an idea.  I thought there was a
difference between plotting ..count.. and ..density.., but in fact it's the
group=1 that makes the first example work.  Both of these behave as one
would hope:

ggplot(df,aes(names,group=1))+geom_histogram()+xlim(Bob,Mary)
ggplot(df,aes(names,..count..,group=1))+geom_histogram()+xlim(Bob,Mary)

...though I admit that I'm not exactly sure why.  Can anyone explain?

Thanks,

-J

[[alternative HTML version deleted]]

__
R-help@r-project.org mailing list
https://stat.ethz.ch/mailman/listinfo/r-help
PLEASE do read the posting guide http://www.R-project.org/posting-guide.html
and provide commented, minimal, self-contained, reproducible code.


[R] Help with Names

2010-05-10 Thread Ravi Ramaswamy
Hi - a newbie question, if someone can please help

I want to change X1, X2,,.to X.1 X.2 etc in the names below.  I am using
the Principal Component Regression function (pcr) and it seems to want it
this way

  datap3.pcr - pcr(water ~ X, 10, data = datap3, Validation =cv)
Error in model.frame.default(formula = water ~ X, data = datap3) :
  invalid type (list) for variable 'X'

--

 names(X)
  [1] X1   X2   X3   X4   X5   X6   X7   X8   X9   X10
X11  X12  X13  X14  X15  X16  X17  X18  X19  X20
 [21] X21  X22  X23  X24  X25  X26  X27  X28  X29  X30
X31  X32  X33  X34  X35  X36  X37  X38  X39  X40
 [41] X41  X42  X43  X44  X45  X46  X47  X48  X49  X50
X51  X52  X53  X54  X55  X56  X57  X58  X59  X60
 [61] X61  X62  X63  X64  X65  X66  X67  X68  X69  X70
X71  X72  X73  X74  X75  X76  X77  X78  X79  X80
 [81] X81  X82  X83  X84  X85  X86  X87  X88  X89  X90
X91  X92  X93  X94  X95  X96  X97  X98  X99  X100
 names(X)[1]
[1] X1
 for(i in 1:100){names(X)[i] - X.i}
 names(X)
  [1] X.i X.i X.i X.i X.i X.i X.i X.i X.i X.i X.i
X.i X.i X.i X.i X.i X.i X.i X.i X.i X.i X.i X.i
X.i
 [25] X.i X.i X.i X.i X.i X.i X.i X.i X.i X.i X.i
X.i X.i X.i X.i X.i X.i X.i X.i X.i X.i X.i X.i
X.i
 [49] X.i X.i X.i X.i X.i X.i X.i X.i X.i X.i X.i
X.i X.i X.i X.i X.i X.i X.i X.i X.i X.i X.i X.i
X.i
 [73] X.i X.i X.i X.i X.i X.i X.i X.i X.i X.i X.i
X.i X.i X.i X.i X.i X.i X.i X.i X.i X.i X.i X.i
X.i
 [97] X.i X.i X.i X.i
 for(i in 1:100){names(X)[i] - X.i}
Error: object 'X.i' not found
 for(i in 1:100){names(X)[i] - X.i}

[[alternative HTML version deleted]]

__
R-help@r-project.org mailing list
https://stat.ethz.ch/mailman/listinfo/r-help
PLEASE do read the posting guide http://www.R-project.org/posting-guide.html
and provide commented, minimal, self-contained, reproducible code.


Re: [R] Help with Names

2010-05-10 Thread jim holtman
Is this what you want:

 x - paste(X, 1:10, sep='')
 x
 [1] X1  X2  X3  X4  X5  X6  X7  X8  X9  X10
 sub(X, X., x)
 [1] X.1  X.2  X.3  X.4  X.5  X.6  X.7  X.8  X.9  X.10



On Mon, May 10, 2010 at 8:53 PM, Ravi Ramaswamy raram...@gmail.com wrote:

 Hi - a newbie question, if someone can please help

 I want to change X1, X2,,.to X.1 X.2 etc in the names below.  I am
 using
 the Principal Component Regression function (pcr) and it seems to want it
 this way

   datap3.pcr - pcr(water ~ X, 10, data = datap3, Validation =cv)
 Error in model.frame.default(formula = water ~ X, data = datap3) :
  invalid type (list) for variable 'X'

 --

  names(X)
  [1] X1   X2   X3   X4   X5   X6   X7   X8   X9   X10
 X11  X12  X13  X14  X15  X16  X17  X18  X19  X20
  [21] X21  X22  X23  X24  X25  X26  X27  X28  X29  X30
 X31  X32  X33  X34  X35  X36  X37  X38  X39  X40
  [41] X41  X42  X43  X44  X45  X46  X47  X48  X49  X50
 X51  X52  X53  X54  X55  X56  X57  X58  X59  X60
  [61] X61  X62  X63  X64  X65  X66  X67  X68  X69  X70
 X71  X72  X73  X74  X75  X76  X77  X78  X79  X80
  [81] X81  X82  X83  X84  X85  X86  X87  X88  X89  X90
 X91  X92  X93  X94  X95  X96  X97  X98  X99  X100
  names(X)[1]
 [1] X1
  for(i in 1:100){names(X)[i] - X.i}
  names(X)
  [1] X.i X.i X.i X.i X.i X.i X.i X.i X.i X.i X.i
 X.i X.i X.i X.i X.i X.i X.i X.i X.i X.i X.i X.i
 X.i
  [25] X.i X.i X.i X.i X.i X.i X.i X.i X.i X.i X.i
 X.i X.i X.i X.i X.i X.i X.i X.i X.i X.i X.i X.i
 X.i
  [49] X.i X.i X.i X.i X.i X.i X.i X.i X.i X.i X.i
 X.i X.i X.i X.i X.i X.i X.i X.i X.i X.i X.i X.i
 X.i
  [73] X.i X.i X.i X.i X.i X.i X.i X.i X.i X.i X.i
 X.i X.i X.i X.i X.i X.i X.i X.i X.i X.i X.i X.i
 X.i
  [97] X.i X.i X.i X.i
  for(i in 1:100){names(X)[i] - X.i}
 Error: object 'X.i' not found
  for(i in 1:100){names(X)[i] - X.i}

[[alternative HTML version deleted]]

 __
 R-help@r-project.org mailing list
 https://stat.ethz.ch/mailman/listinfo/r-help
 PLEASE do read the posting guide
 http://www.R-project.org/posting-guide.htmlhttp://www.r-project.org/posting-guide.html
 and provide commented, minimal, self-contained, reproducible code.




-- 
Jim Holtman
Cincinnati, OH
+1 513 646 9390

What is the problem that you are trying to solve?

[[alternative HTML version deleted]]

__
R-help@r-project.org mailing list
https://stat.ethz.ch/mailman/listinfo/r-help
PLEASE do read the posting guide http://www.R-project.org/posting-guide.html
and provide commented, minimal, self-contained, reproducible code.


Re: [R] Help with Names

2010-05-10 Thread Ravi Ramaswamy
Jim - thanks.

It worked, however pcr program is still not accepting it there is a
sample yarn data loaded and I am comparing.  My data looks like this

  X1  X2  X3  X4  X5  X6  X7
X8  X9 X10 X11 X12 X13 X14 X15 X16 X17
1Kalle   2.36038 2.35396 2.34772 2.34167 2.33587 2.33036 2.32517 2.32033
2.31583 2.31168 2.30791 2.30450 2.30150 2.29891 2.29680 2.29523 2.29421

and yarn data looks like this

  NIR.1   NIR.2   NIR.3   NIR.4   NIR.5   NIR.6   NIR.7   NIR.8   NIR.9
NIR.10  NIR.11  NIR.12  NIR.13  NIR.14  NIR.15  NIR.16  NIR.17  NIR.18
1   3.06630 3.08610 3.10790 3.09720 2.99790 2.82730 2.62330 2.40390 2.19310
2.00580 1.83790 1.69070 1.57770 1.50330 1.43810 1.33730 1.22060 1.14100

So, I thought I could convert X1 to X.1 and it would work.  The names seem
different.  Not sure how to make the change

 names(yarn)
[1] NIR density train

but

 names(X)
  [1] X.1   X.2   X.3   X.4   X.5   X.6   X.7   X.8
X.9   X.10  X.11  X.12  X.13  X.14  X.15  X.16  X.17
X.18

On Mon, May 10, 2010 at 9:05 PM, jim holtman jholt...@gmail.com wrote:

 Is this what you want:

  x - paste(X, 1:10, sep='')

  x
  [1] X1  X2  X3  X4  X5  X6  X7  X8  X9  X10
  sub(X, X., x)
  [1] X.1  X.2  X.3  X.4  X.5  X.6  X.7  X.8  X.9  X.10
 


 On Mon, May 10, 2010 at 8:53 PM, Ravi Ramaswamy raram...@gmail.comwrote:

 Hi - a newbie question, if someone can please help

 I want to change X1, X2,,.to X.1 X.2 etc in the names below.  I am
 using
 the Principal Component Regression function (pcr) and it seems to want it
 this way

   datap3.pcr - pcr(water ~ X, 10, data = datap3, Validation =cv)
 Error in model.frame.default(formula = water ~ X, data = datap3) :
  invalid type (list) for variable 'X'

 --

  names(X)
  [1] X1   X2   X3   X4   X5   X6   X7   X8   X9   X10
 X11  X12  X13  X14  X15  X16  X17  X18  X19  X20
  [21] X21  X22  X23  X24  X25  X26  X27  X28  X29  X30
 X31  X32  X33  X34  X35  X36  X37  X38  X39  X40
  [41] X41  X42  X43  X44  X45  X46  X47  X48  X49  X50
 X51  X52  X53  X54  X55  X56  X57  X58  X59  X60
  [61] X61  X62  X63  X64  X65  X66  X67  X68  X69  X70
 X71  X72  X73  X74  X75  X76  X77  X78  X79  X80
  [81] X81  X82  X83  X84  X85  X86  X87  X88  X89  X90
 X91  X92  X93  X94  X95  X96  X97  X98  X99  X100
  names(X)[1]
 [1] X1
  for(i in 1:100){names(X)[i] - X.i}
  names(X)
  [1] X.i X.i X.i X.i X.i X.i X.i X.i X.i X.i X.i
 X.i X.i X.i X.i X.i X.i X.i X.i X.i X.i X.i X.i
 X.i
  [25] X.i X.i X.i X.i X.i X.i X.i X.i X.i X.i X.i
 X.i X.i X.i X.i X.i X.i X.i X.i X.i X.i X.i X.i
 X.i
  [49] X.i X.i X.i X.i X.i X.i X.i X.i X.i X.i X.i
 X.i X.i X.i X.i X.i X.i X.i X.i X.i X.i X.i X.i
 X.i
  [73] X.i X.i X.i X.i X.i X.i X.i X.i X.i X.i X.i
 X.i X.i X.i X.i X.i X.i X.i X.i X.i X.i X.i X.i
 X.i
  [97] X.i X.i X.i X.i
  for(i in 1:100){names(X)[i] - X.i}
 Error: object 'X.i' not found
  for(i in 1:100){names(X)[i] - X.i}

[[alternative HTML version deleted]]

 __
 R-help@r-project.org mailing list
 https://stat.ethz.ch/mailman/listinfo/r-help
 PLEASE do read the posting guide
 http://www.R-project.org/posting-guide.htmlhttp://www.r-project.org/posting-guide.html
 and provide commented, minimal, self-contained, reproducible code.




 --
 Jim Holtman
 Cincinnati, OH
 +1 513 646 9390

 What is the problem that you are trying to solve?


[[alternative HTML version deleted]]

__
R-help@r-project.org mailing list
https://stat.ethz.ch/mailman/listinfo/r-help
PLEASE do read the posting guide http://www.R-project.org/posting-guide.html
and provide commented, minimal, self-contained, reproducible code.


Re: [R] Help with Names

2010-05-10 Thread jim holtman
You need to provide an 'str(yarn)' so we can see what the structure is.  I
don't know what the 'pcr' program is expecting, but it looks like from what
you have provided that 'yarn' might be a dataframe and X is a vector.
Look at the documentation for pcr and see what it expects.

On Mon, May 10, 2010 at 9:08 PM, Ravi Ramaswamy raram...@gmail.com wrote:

 Jim - thanks.

 It worked, however pcr program is still not accepting it there is a
 sample yarn data loaded and I am comparing.  My data looks like this


   X1  X2  X3  X4  X5  X6  X7
 X8  X9 X10 X11 X12 X13 X14 X15 X16 X17
 1Kalle   2.36038 2.35396 2.34772 2.34167 2.33587 2.33036 2.32517 2.32033
 2.31583 2.31168 2.30791 2.30450 2.30150 2.29891 2.29680 2.29523 2.29421

 and yarn data looks like this

   NIR.1   NIR.2   NIR.3   NIR.4   NIR.5   NIR.6   NIR.7   NIR.8
 NIR.9  NIR.10  NIR.11  NIR.12  NIR.13  NIR.14  NIR.15  NIR.16  NIR.17
 NIR.18
 1   3.06630 3.08610 3.10790 3.09720 2.99790 2.82730 2.62330 2.40390 2.19310
 2.00580 1.83790 1.69070 1.57770 1.50330 1.43810 1.33730 1.22060 1.14100

 So, I thought I could convert X1 to X.1 and it would work.  The names seem
 different.  Not sure how to make the change

  names(yarn)
 [1] NIR density train

 but

  names(X)
   [1] X.1   X.2   X.3   X.4   X.5   X.6   X.7   X.8
 X.9   X.10  X.11  X.12  X.13  X.14  X.15  X.16  X.17
 X.18


 On Mon, May 10, 2010 at 9:05 PM, jim holtman jholt...@gmail.com wrote:

 Is this what you want:

  x - paste(X, 1:10, sep='')

  x
  [1] X1  X2  X3  X4  X5  X6  X7  X8  X9  X10
  sub(X, X., x)
  [1] X.1  X.2  X.3  X.4  X.5  X.6  X.7  X.8  X.9  X.10
 


   On Mon, May 10, 2010 at 8:53 PM, Ravi Ramaswamy raram...@gmail.comwrote:

  Hi - a newbie question, if someone can please help

 I want to change X1, X2,,.to X.1 X.2 etc in the names below.  I am
 using
 the Principal Component Regression function (pcr) and it seems to want it
 this way

   datap3.pcr - pcr(water ~ X, 10, data = datap3, Validation =cv)
 Error in model.frame.default(formula = water ~ X, data = datap3) :
  invalid type (list) for variable 'X'

 --

  names(X)
  [1] X1   X2   X3   X4   X5   X6   X7   X8   X9   X10
 X11  X12  X13  X14  X15  X16  X17  X18  X19  X20
  [21] X21  X22  X23  X24  X25  X26  X27  X28  X29
  X30
 X31  X32  X33  X34  X35  X36  X37  X38  X39  X40
  [41] X41  X42  X43  X44  X45  X46  X47  X48  X49
  X50
 X51  X52  X53  X54  X55  X56  X57  X58  X59  X60
  [61] X61  X62  X63  X64  X65  X66  X67  X68  X69
  X70
 X71  X72  X73  X74  X75  X76  X77  X78  X79  X80
  [81] X81  X82  X83  X84  X85  X86  X87  X88  X89
  X90
 X91  X92  X93  X94  X95  X96  X97  X98  X99  X100
  names(X)[1]
 [1] X1
  for(i in 1:100){names(X)[i] - X.i}
  names(X)
  [1] X.i X.i X.i X.i X.i X.i X.i X.i X.i X.i X.i
 X.i X.i X.i X.i X.i X.i X.i X.i X.i X.i X.i X.i
 X.i
  [25] X.i X.i X.i X.i X.i X.i X.i X.i X.i X.i X.i
 X.i X.i X.i X.i X.i X.i X.i X.i X.i X.i X.i X.i
 X.i
  [49] X.i X.i X.i X.i X.i X.i X.i X.i X.i X.i X.i
 X.i X.i X.i X.i X.i X.i X.i X.i X.i X.i X.i X.i
 X.i
  [73] X.i X.i X.i X.i X.i X.i X.i X.i X.i X.i X.i
 X.i X.i X.i X.i X.i X.i X.i X.i X.i X.i X.i X.i
 X.i
  [97] X.i X.i X.i X.i
  for(i in 1:100){names(X)[i] - X.i}
 Error: object 'X.i' not found
  for(i in 1:100){names(X)[i] - X.i}

[[alternative HTML version deleted]]

 __
 R-help@r-project.org mailing list
 https://stat.ethz.ch/mailman/listinfo/r-help
 PLEASE do read the posting guide
 http://www.R-project.org/posting-guide.htmlhttp://www.r-project.org/posting-guide.html
 and provide commented, minimal, self-contained, reproducible code.




 --
 Jim Holtman
 Cincinnati, OH
 +1 513 646 9390

 What is the problem that you are trying to solve?





-- 
Jim Holtman
Cincinnati, OH
+1 513 646 9390

What is the problem that you are trying to solve?

[[alternative HTML version deleted]]

__
R-help@r-project.org mailing list
https://stat.ethz.ch/mailman/listinfo/r-help
PLEASE do read the posting guide http://www.R-project.org/posting-guide.html
and provide commented, minimal, self-contained, reproducible code.


Re: [R] Help with Names

2010-05-10 Thread Ravi Ramaswamy
There is definitely a difference.  The pcr program is Principal Component
Regression, but document says

The formula argument should be a symbolic formula of the form response ~
terms, where response is the name of the response vector or matrix (for
multi-response models) and terms is the name of one or more predictor
matrices, usually separated by +, e.g., water ~ FTIR or y ~ X + Z. See lm
for a detailed description. The named variables should exist in the supplied
data data frame or in the global environment. 

Not sure what to do next ...

 str(yarn)
'data.frame':28 obs. of  3 variables:
 $ NIR: num [1:28, 1:268] 3.07 3.07 3.08 3.08 3.1 ...
  ..- attr(*, dimnames)=List of 2
  .. ..$ : NULL
  .. ..$ : NULL
 $ density: num  100 80.2 79.5 60.8 60 ...
 $ train  : logi  TRUE TRUE TRUE TRUE TRUE TRUE ...
 str(X)
'data.frame':79 obs. of  100 variables:
 $ X.1  : num  2.36 2.36 2.15 2.22 2.3 ...
 $ X.2  : num  2.35 2.36 2.14 2.22 2.3 ...
 $ X.3  : num  2.35 2.35 2.14 2.21 2.29 ...
 $ X.4  : num  2.34 2.34 2.13 2.21 2.28 ...

On Mon, May 10, 2010 at 9:19 PM, jim holtman jholt...@gmail.com wrote:

 You need to provide an 'str(yarn)' so we can see what the structure is.  I
 don't know what the 'pcr' program is expecting, but it looks like from what
 you have provided that 'yarn' might be a dataframe and X is a vector.
 Look at the documentation for pcr and see what it expects.


 On Mon, May 10, 2010 at 9:08 PM, Ravi Ramaswamy raram...@gmail.comwrote:

 Jim - thanks.

 It worked, however pcr program is still not accepting it there is a
 sample yarn data loaded and I am comparing.  My data looks like this


   X1  X2  X3  X4  X5  X6  X7
 X8  X9 X10 X11 X12 X13 X14 X15 X16 X17
 1Kalle   2.36038 2.35396 2.34772 2.34167 2.33587 2.33036 2.32517 2.32033
 2.31583 2.31168 2.30791 2.30450 2.30150 2.29891 2.29680 2.29523 2.29421

 and yarn data looks like this

   NIR.1   NIR.2   NIR.3   NIR.4   NIR.5   NIR.6   NIR.7   NIR.8
 NIR.9  NIR.10  NIR.11  NIR.12  NIR.13  NIR.14  NIR.15  NIR.16  NIR.17
 NIR.18
 1   3.06630 3.08610 3.10790 3.09720 2.99790 2.82730 2.62330 2.40390
 2.19310 2.00580 1.83790 1.69070 1.57770 1.50330 1.43810 1.33730 1.22060
 1.14100

 So, I thought I could convert X1 to X.1 and it would work.  The names seem
 different.  Not sure how to make the change

  names(yarn)
 [1] NIR density train

 but

  names(X)
   [1] X.1   X.2   X.3   X.4   X.5   X.6   X.7   X.8
 X.9   X.10  X.11  X.12  X.13  X.14  X.15  X.16  X.17
 X.18


 On Mon, May 10, 2010 at 9:05 PM, jim holtman jholt...@gmail.com wrote:

 Is this what you want:

  x - paste(X, 1:10, sep='')

  x
  [1] X1  X2  X3  X4  X5  X6  X7  X8  X9  X10
  sub(X, X., x)
  [1] X.1  X.2  X.3  X.4  X.5  X.6  X.7  X.8  X.9
 X.10
 


   On Mon, May 10, 2010 at 8:53 PM, Ravi Ramaswamy raram...@gmail.comwrote:

  Hi - a newbie question, if someone can please help

 I want to change X1, X2,,.to X.1 X.2 etc in the names below.  I am
 using
 the Principal Component Regression function (pcr) and it seems to want
 it
 this way

   datap3.pcr - pcr(water ~ X, 10, data = datap3, Validation =cv)
 Error in model.frame.default(formula = water ~ X, data = datap3) :
  invalid type (list) for variable 'X'

 --

  names(X)
  [1] X1   X2   X3   X4   X5   X6   X7   X8   X9
 X10
 X11  X12  X13  X14  X15  X16  X17  X18  X19  X20
  [21] X21  X22  X23  X24  X25  X26  X27  X28  X29
  X30
 X31  X32  X33  X34  X35  X36  X37  X38  X39  X40
  [41] X41  X42  X43  X44  X45  X46  X47  X48  X49
  X50
 X51  X52  X53  X54  X55  X56  X57  X58  X59  X60
  [61] X61  X62  X63  X64  X65  X66  X67  X68  X69
  X70
 X71  X72  X73  X74  X75  X76  X77  X78  X79  X80
  [81] X81  X82  X83  X84  X85  X86  X87  X88  X89
  X90
 X91  X92  X93  X94  X95  X96  X97  X98  X99  X100
  names(X)[1]
 [1] X1
  for(i in 1:100){names(X)[i] - X.i}
  names(X)
  [1] X.i X.i X.i X.i X.i X.i X.i X.i X.i X.i X.i
 X.i X.i X.i X.i X.i X.i X.i X.i X.i X.i X.i X.i
 X.i
  [25] X.i X.i X.i X.i X.i X.i X.i X.i X.i X.i X.i
 X.i X.i X.i X.i X.i X.i X.i X.i X.i X.i X.i X.i
 X.i
  [49] X.i X.i X.i X.i X.i X.i X.i X.i X.i X.i X.i
 X.i X.i X.i X.i X.i X.i X.i X.i X.i X.i X.i X.i
 X.i
  [73] X.i X.i X.i X.i X.i X.i X.i X.i X.i X.i X.i
 X.i X.i X.i X.i X.i X.i X.i X.i X.i X.i X.i X.i
 X.i
  [97] X.i X.i X.i X.i
  for(i in 1:100){names(X)[i] - X.i}
 Error: object 'X.i' not found
  for(i in 1:100){names(X)[i] - X.i}

[[alternative HTML version deleted]]

 __
 R-help@r-project.org mailing list
 https://stat.ethz.ch/mailman/listinfo/r-help
 PLEASE do read the posting guide
 http://www.R-project.org/posting-guide.htmlhttp://www.r-project.org/posting-guide.html
 and provide commented, minimal, self-contained, reproducible code.




 --
 Jim Holtman
 Cincinnati, OH
 +1 513 646 9390

 What is the problem that you are trying to solve?





 --
 Jim Holtman
 Cincinnati, OH
 +1 513 646 9390

 What 

[R] kernel density to smooth plots

2010-05-10 Thread Roslina Zakaria

Hi r-sers,
I have a data of relative frequencies for the interval of 0-20, 
20-40,...380-400.  I would like the two data on the same graph using the same 
x-axis label.  My question is how to get a smooth curve using kernel density 
code if it possible for this data.
 
 cbind(rel_obs,rel_gen)
  rel_obs rel_gen
 [1,] 0.0  0.
 [2,] 0.092534175  0.0712
 [3,] 0.105152471  0.1092
 [4,] 0.095688749  0.1264
 [5,] 0.107255521  0.1143
 [6,] 0.098843323  0.1063
 [7,] 0.085173502  0.0878
 [8,] 0.084121977  0.0727
 [9,] 0.064143007  0.0637
[10,] 0.056782334  0.0475
[11,] 0.048370137  0.0402
[12,] 0.041009464  0.0314
[13,] 0.021030494  0.0269
[14,] 0.029442692  0.0235
[15,] 0.018927445  0.0183
[16,] 0.015772871  0.0134
[17,] 0.009463722  0.0100
[18,] 0.007360673  0.0089
[19,] 0.005257624  0.0072
[20,] 0.004206099  0.0033
[21,] 0.004206099  0.0034
[22,] 0.003154574  0.0030
[23,] 0.002103049  0.0024
[24,] 0.0  0.0018
[25,] 0.0  0.0015
[26,] 0.0  0.0019
[27,] 0.0  0.0011
[28,] 0.0  0.0006
[29,] 0.0  0.0009
[30,] 0.0  0.0006
[31,] 0.0  0.0003
[32,] 0.0  0.0001
[33,] 0.0  0.
[34,] 0.0  0.0001
[35,] 0.0  0.
[36,] 0.0  0.
[37,] 0.0  0.0001
[38,] 0.0  0.
[39,] 0.0  0.
[40,] 0.0  0.
[41,] 0.0  0.
 
Thank you.



  
[[alternative HTML version deleted]]

__
R-help@r-project.org mailing list
https://stat.ethz.ch/mailman/listinfo/r-help
PLEASE do read the posting guide http://www.R-project.org/posting-guide.html
and provide commented, minimal, self-contained, reproducible code.


[R] Advice needed on awkward tables

2010-05-10 Thread Greg Orm
Dear r-help list members,

I am quite new to R, and hope to seek advice from you about a problem I have
been cracking my head over. Apologies if this seems like a simple problem.

I have essentially two tables. The first (Table A) is a standard patient
clinicopathological data table, where rows correspond to patient IDs and
columns correspond to clinical features. Records in this table are stored as
1 or 0 (denoting presence). An example is provided below.

The second (Table B) is a table that represents a 'key' to Table A. This
Table B has a category field, as well as a feature field which links to the
Table B. Unfortunately, this is a one-to-many relationship, and the numbers
in the feature field represent the respective columns in Table A, delimited
by semicolons. So in the example below, I need to collapse the data in Table
B into a table with nrow equivalent to the number of categories and ncol =
number of patients. The collapsing of each categoriy, will be based on a
Boolean OR, or the equivalent ANY in R (so long as 1 of the features is
true, the resulting outcome will be true)

data.table.a -
matrix(data=round(runif(100)),nrow=10,ncol=10,dimnames=list(paste(Patient,1:10),paste(Feature,1:10)))
data.table.b - data.frame
(ID=c(1,2,3,4,5,6,7),CATEGORY=c(1,2,3,3,4,5,5),FEATURE=c(9,3;5,7,4,6;10,1;2,8))

In the example tables above, we hope to collapse the features by category -
so the final desired output will be a total of 10 patients as rows, and a
total of 5 categories as columns. (after collapsing the features by a
Boolean OR). (i.e. if any of the features in the category are present, it
will be a TRUE).

I apologize for the apparently awkward table, but this is what I had to
start with. I tried expanding data.table.b$FEATURE using strsplit, which
resulted in a list, and then I got stuck there for a long time.

Thanks for any help.

Greg

[[alternative HTML version deleted]]

__
R-help@r-project.org mailing list
https://stat.ethz.ch/mailman/listinfo/r-help
PLEASE do read the posting guide http://www.R-project.org/posting-guide.html
and provide commented, minimal, self-contained, reproducible code.


[R] dbSendQuery with R variables

2010-05-10 Thread Jonathan Greenberg
Rhelpers:

I'd like to modify this RSQLite statement:

rs_stations-dbSendQuery(con_stations, select * from stations)

so that stations is actually an R variable, e.g.:

stations=c(stationA,stationB)

How would I modify the above statement to query from stations[[1]]
(aka stationA)?

--j

__
R-help@r-project.org mailing list
https://stat.ethz.ch/mailman/listinfo/r-help
PLEASE do read the posting guide http://www.R-project.org/posting-guide.html
and provide commented, minimal, self-contained, reproducible code.


  1   2   >